LE5 Gyne Flashcards

1
Q
  1. Identify the histologic diagnosis in the provided image (Image 14).
    A. Simple hyperplasia with atypia
    B. Complex hyperplasia without atypia
    C. Endometrial carcinoma
    D. Endometrial stromal sarcoma
A

A. Simple hyperplasia with atypia

πŸ”Ž Rationale:

Simple hyperplasia with atypia shows increased gland-to-stroma ratio but retains glandular architecture.
Atypia refers to nuclear changes such as enlarged, irregular nuclei.
Progression risk to endometrial carcinoma is lower than complex hyperplasia with atypia.
πŸ“Œ Source: Histologic classifications of endometrial hyperplasia

How well did you know this?
1
Not at all
2
3
4
5
Perfectly
2
Q

Identify the histologic diagnosis in the provided image (Image 15).
A. Complex hyperplasia with atypia
B. Simple hyperplasia without atypia
C. Endometrial carcinoma
D. Endometrial stromal sarcoma

A

A. Complex hyperplasia with atypia

πŸ”Ž Rationale:

Complex hyperplasia with atypia is characterized by irregularly shaped glands with little intervening stroma and nuclear atypia.
It has a high risk of progression to endometrial carcinoma (~29-42%).
πŸ“Œ Source: Histologic classifications of endometrial hyperplasia

How well did you know this?
1
Not at all
2
3
4
5
Perfectly
3
Q
  1. Most common high-risk HPV types responsible for cervical cancer are?
    A. HPV 11 and HPV 18
    B. HPV 6 and HPV 16
    C. HPV 16 and HPV 18
    D. HPV 31 and HPV 33
A

C. HPV 16 and HPV 18

πŸ“Œ Rationale: HPV 16 and 18 are the most oncogenic types and are responsible for most HPV-related cancers. There are about 14 high-risk HPV types, but these two account for the majority of cervical cancer cases. (Document Reference: Cervical Cancer - Pathogenesis section)

How well did you know this?
1
Not at all
2
3
4
5
Perfectly
4
Q
  1. A small percentage of women infected with HPV will develop cervical cancer within how many years?
    A. 5-10 years
    B. 10-15 years
    C. 15-20 years
    D. 20-25 years
A

C. 15-20 years

πŸ“Œ Rationale: Persistent HPV infection is a necessary cause of cervical cancer, but it takes 10-20 years to progress in most cases. The progression is faster (as short as 5 years) in immunocompromised individuals, such as those with HIV. (Document Reference: Pathogenesis section - HPV infection remains for 10-20 years before becoming cancer)

How well did you know this?
1
Not at all
2
3
4
5
Perfectly
5
Q
  1. The most common early symptom of cervical cancer is?
    A. Pelvic pain
    B. Unusual vaginal discharge
    C. Weight loss
    D. Lower back pain
A

B. Unusual vaginal discharge

πŸ“Œ Rationale: The most common early signs of cervical cancer include abnormal vaginal discharge, abnormal vaginal bleeding, and heavier & longer menstrual cycles. Pelvic pain and weight loss are late-stage symptoms. (Document Reference: Clinical Manifestation section - Early/Past Early Symptoms)

How well did you know this?
1
Not at all
2
3
4
5
Perfectly
6
Q
  1. Which of the following is TRUE regarding cervical cancer?
    A. It is the 2nd most common malignancy in women
    B. It is the 3rd most common malignancy among women
    C. It is the most common malignancy in women
    D. It is the least common gynecologic malignancy
A

B. It is the 3rd most common malignancy among women

πŸ“Œ Rationale: Cervical cancer remains the 3rd most common malignancy among women worldwide and the most common gynecologic malignancy. (Document Reference: Epidemiology section, β€œCervical cancer remains the 3rd most common malignancy among women and most common gynecologic malignancy.”)

How well did you know this?
1
Not at all
2
3
4
5
Perfectly
7
Q
  1. Which of the following is NOT a risk factor for cervical cancer?
    A. Smoking
    B. No Pap smear
    C. Nulliparity
    D. Early coitarche
A

C. Nulliparity

πŸ“Œ Rationale: High parity (multiple childbirths) is a risk factor for cervical cancer because it exposes the transformation zone of the cervix to HPV for an extended period. Nulliparity (having no children) is not a known risk factor. (Document Reference: Risk Factors Table - High Parity is listed as a risk, Nulliparity is NOT mentioned.)

How well did you know this?
1
Not at all
2
3
4
5
Perfectly
8
Q
  1. A 22-year-old woman comes for cervical cancer screening. What is the recommended test?
    A. HPV test
    B. HPV + Pap smear
    C. Pap smear
    D. Acetowhite test
A

C. Pap smear

πŸ“Œ Rationale: Women aged 21-29 should undergo Pap smear screening every 3 years. HPV testing is not recommended for women under 30 because HPV infection is common and often transient in this age group. (Document Reference: Screening Guidelines Table - β€œ21-29 y/o β†’ Pap every 3 years”)

How well did you know this?
1
Not at all
2
3
4
5
Perfectly
9
Q
  1. A 35-year-old woman has a Pap smear result showing HSIL (High-Grade Squamous Intraepithelial Lesion). What is the next best step?
    A. Repeat Pap smear after 6 months
    B. Cervical biopsy
    C. Colposcopy with possible cervical biopsy
    D. HPV test
A

C. Colposcopy with possible cervical biopsy

πŸ“Œ Rationale: HSIL (CIN 2, 3) is a high-risk precancerous lesion that requires further evaluation with colposcopy and possible biopsy. Pap smear should not be repeated, as HSIL requires immediate follow-up. (Document Reference: Management of Premalignant Lesions Table - HSIL β†’ β€œColposcopy with cervical biopsy”)

How well did you know this?
1
Not at all
2
3
4
5
Perfectly
10
Q
  1. What is the recommended cervical cancer screening method for women aged 30 years and above?
    A. Pap smear
    B. Pap smear with colposcopy
    C. HPV test with Pap smear
    D. HPV test with colposcopy
A

C. HPV test with Pap smear

πŸ“Œ Rationale: For women aged 30-65, the preferred screening method is co-testing (Pap smear + HPV test every 5 years). If HPV testing is not available, Pap smear alone every 3 years is also acceptable. (Document Reference: Screening Guidelines Table - β€œ30-65 y/o β†’ Pap + HPV every 5 years OR Cytology alone every 3 years”)

How well did you know this?
1
Not at all
2
3
4
5
Perfectly
11
Q
  1. A woman underwent colposcopy for an abnormal HSIL result. What area should be biopsied?
    A. Normal squamous epithelium
    B. Acetowhitening area
    C. Transformation zone only
    D. Any random cervical site
A

B. Acetowhitening area

πŸ“Œ Rationale: During colposcopy, acetic acid is applied to the cervix to highlight abnormal cells. Dysplastic cells with large nuclei reflect light and appear white (acetowhite lesions), which should be biopsied. (Document Reference: Colposcopy section - β€œAcetic acid washes away mucus and highlights abnormal areas by staining them white.”)

How well did you know this?
1
Not at all
2
3
4
5
Perfectly
12
Q
  1. A 27-year-old HIV-positive woman asks how long she should continue Pap smear screening. What is the correct advice?
    A. Every 3 years
    B. Every year
    C. Throughout her lifetime
    D. Stop at age 65 if three negative results are documented
A

C. Throughout her lifetime

πŸ“Œ Rationale: HIV-positive women should undergo lifetime cervical cancer screening. They should have Pap smears twice in the first year after HIV diagnosis, then annually if results are normal. This is because HIV leads to immunosuppression, making HPV infection more persistent and increasing cervical cancer risk. (Document Reference: HIV Screening Guidelines - β€œScreening should continue throughout a woman’s lifetime.”)

How well did you know this?
1
Not at all
2
3
4
5
Perfectly
13
Q
  1. A woman who was vaccinated for HPV has not had a Pap smear in the past 5 years. What should she do?
    A. No screening needed since she is vaccinated
    B. Pap smear only
    C. Pap smear + HPV test (Co-testing)
    D. Colposcopy
A

C. Pap smear + HPV test (Co-testing)

πŸ“Œ Rationale: Even if a woman is vaccinated against HPV, she still needs cervical cancer screening. The preferred method for women 30-65 years old is co-testing (Pap smear + HPV test every 5 years). Vaccination does not eliminate the need for screening. (Document Reference: Screening Guidelines Table - β€œPap + HPV (co-testing preferred), every 5 years OR Cytology alone every 3 years.”)

How well did you know this?
1
Not at all
2
3
4
5
Perfectly
14
Q
  1. What is the recommended Pap smear guideline for HIV-positive women?
    A. Pap smear every 3 years
    B. Pap smear every 5 years
    C. Lifetime Pap smear screening
    D. Stop at age 65 if negative for the past 10 years
A

C. Lifetime Pap smear screening

πŸ“Œ Rationale: HIV-positive women require lifetime cervical cancer screening. Screening should begin within 1 year of first sexual contact or by age 21, whichever comes first. (Document Reference: HIV Screening Guidelines - β€œScreening should continue throughout a woman’s lifetime.”)

How well did you know this?
1
Not at all
2
3
4
5
Perfectly
15
Q
  1. A cervical biopsy shows atypical cells with the presence of koilocytes. What is the diagnosis?
    A. LSIL (CIN 1)
    B. HSIL (CIN 2)
    C. HSIL (CIN 3)
    D. Invasive Squamous Cell Carcinoma
A

A. LSIL (CIN 1)

πŸ“Œ Rationale: Koilocytes are hallmark cells of HPV infection and are commonly seen in Low-Grade Squamous Intraepithelial Lesion (LSIL), also known as CIN 1. CIN 1 is mild dysplasia and often resolves spontaneously but requires follow-up. (Document Reference: Premalignant Lesions Table - β€œLSIL (CIN 1) is characterized by koilocytosis and mild dysplasia.”)

How well did you know this?
1
Not at all
2
3
4
5
Perfectly
16
Q
  1. A patient had a Pap smear that revealed HSIL. What is the next step in management?
    A. Repeat Pap smear after 6 months
    B. Colposcopy only
    C. Excisional procedure
    D. Watchful waiting
A

C. Excisional procedure

πŸ“Œ Rationale: High-Grade Squamous Intraepithelial Lesion (HSIL) requires excisional procedures such as LEEP (Loop Electrosurgical Excision Procedure) or Cold Knife Conization to remove the affected area and prevent progression to invasive carcinoma. (Document Reference: Management of Premalignant Lesions - β€œHSIL requires excisional procedures.”)

How well did you know this?
1
Not at all
2
3
4
5
Perfectly
17
Q
  1. What is the most common histologic type of cervical cancer?
    A. Squamous cell carcinoma
    B. Adenocarcinoma
    C. Neuroendocrine carcinoma
    D. Small cell carcinoma
A

A. Squamous cell carcinoma

πŸ“Œ Rationale: Squamous cell carcinoma (SCC) accounts for about 70% of cervical cancer cases. It can be keratinizing or non-keratinizing. The second most common type is adenocarcinoma (25%), which arises from endocervical glandular cells. (Document Reference: Histologic Types - β€œMost common: Squamous (70%)”)

How well did you know this?
1
Not at all
2
3
4
5
Perfectly
18
Q
  1. A 38-year-old woman presents with abnormal bleeding and a barrel-shaped cervix. What is the most likely diagnosis?
    A. Squamous cell carcinoma
    B. Adenocarcinoma
    C. Endometrial cancer
    D. Leiomyosarcoma
A

B. Adenocarcinoma

πŸ“Œ Rationale: A barrel-shaped cervix is characteristic of cervical adenocarcinoma. Unlike SCC, adenocarcinoma grows endophytically, meaning it expands inside the cervix, making it harder to detect on physical exam. (Document Reference: Histologic Types - β€œAdenocarcinoma is often occult and may present as a barrel-shaped cervix.”)

How well did you know this?
1
Not at all
2
3
4
5
Perfectly
19
Q
  1. A patient undergoing colposcopy is found to have pelvic lymph node involvement. What is the correct FIGO stage?
    A. Stage IIIB
    B. Stage IIIC1
    C. Stage IVA
    D. Stage IVB
A

B. Stage IIIC1

πŸ“Œ Rationale: Stage IIIC1 indicates pelvic lymph node metastasis without para-aortic node involvement. If para-aortic lymph nodes were involved, it would be Stage IIIC2. (Document Reference: FIGO Staging Table - β€œIIIC1 = Pelvic lymph node involvement.”)

How well did you know this?
1
Not at all
2
3
4
5
Perfectly
20
Q
  1. A patient undergoing colposcopy has blood-streaked stool. What is the correct FIGO stage?
    A. Stage IIB
    B. Stage IIIC2
    C. Stage IVA
    D. Stage IVB
A

C. Stage IVA

πŸ“Œ Rationale: Stage IVA cervical cancer means the tumor has spread to adjacent pelvic organs, such as the bladder or rectum. Blood-streaked stool suggests rectal invasion. If the cancer had spread to distant organs, it would be Stage IVB. (Document Reference: FIGO Staging Table - β€œIVA = Spread to adjacent pelvic organs.”)

How well did you know this?
1
Not at all
2
3
4
5
Perfectly
21
Q
  1. What is the treatment of cervical cancer that can be done at all stages?
    A. Surgery
    B. Concurrent chemoradiation
    C. Palliative chemotherapy
    D. Radiation therapy alone
A

B. Concurrent chemoradiation

πŸ“Œ Rationale: Concurrent chemoradiation (radiation + platinum-based chemotherapy) is the standard treatment for cervical cancer from Stage I to Stage IV. Surgery is usually only an option for very early-stage disease (IA, IB1, IIA1), and even then, concurrent chemoradiation may still be preferred. (Document Reference: Treatment Table - β€œConcurrent chemoradiation can be done in Stages I-IV.”)

How well did you know this?
1
Not at all
2
3
4
5
Perfectly
22
Q
  1. What is the definitive management for vulvar intraepithelial neoplasia (VIN)?
    A. Topical imiquimod
    B. Chemotherapy
    C. Surgery
    D. Radiation therapy
A

C. Surgery

πŸ“Œ Rationale: Surgical excision is the definitive treatment for VIN. While topical treatments like imiquimod can be used for some cases, surgery (wide local excision, skinning vulvectomy) is the gold standard. (Document Reference: VIN Management Table - β€œSurgical excision is the mainstay of treatment.”)

How well did you know this?
1
Not at all
2
3
4
5
Perfectly
23
Q
  1. A 55-year-old woman presents with atypical cells involving more than 2/3 of the vaginal epithelium. What is the most likely diagnosis?
    A. VAIN 1
    B. VAIN 2
    C. VAIN 3
    D. Vaginal squamous cell carcinoma
A

C. VAIN 3

πŸ“Œ Rationale: Vaginal intraepithelial neoplasia (VAIN) is classified based on the extent of epithelial involvement.

VAIN 1 β†’ Involves lower 1/3 of the epithelium
VAIN 2 β†’ Involves 2/3 of the epithelium
VAIN 3 β†’ Involves more than 2/3 of the epithelium and is considered carcinoma in situ. (Document Reference: VAIN Staging Table)

How well did you know this?
1
Not at all
2
3
4
5
Perfectly
24
Q
  1. Which of the following is NOT a risk factor for vaginal intraepithelial neoplasia (VAIN)?
    A. HPV infection
    B. Smoking
    C. Multiple sexual partners
    D. Douching
A

D. Douching

πŸ“Œ Rationale: HPV infection, smoking, and multiple sexual partners are established risk factors for VAIN. Douching is NOT a recognized risk factor for VAIN or cervical cancer. (Document Reference: Risk Factors for VAIN - β€œHPV, smoking, and sexual history are the main risks.”)

How well did you know this?
1
Not at all
2
3
4
5
Perfectly
25
Q
  1. An 83-year-old patient presents with a dark pigmented lesion on the vulva. What is the most likely diagnosis?
    A. Squamous cell carcinoma
    B. Melanoma
    C. Paget’s disease
    D. Lichen planus
A

B. Melanoma

πŸ“Œ Rationale: Melanoma is the second most common malignant neoplasm of the vulva and often presents as a dark pigmented lesion. It is more common in older patients (age >70) and has a poor prognosis. (Document Reference: Vulvar Cancer - β€œMelanoma is rare but common in older patients.”)

How well did you know this?
1
Not at all
2
3
4
5
Perfectly
26
Q
  1. A 65-year-old woman presents with a bulging mass near the vaginal opening. What is the most likely diagnosis?
    A. Bartholin’s gland cyst
    B. Bartholin’s gland carcinoma
    C. Vulvar abscess
    D. Lipoma
A

B. Bartholin’s gland carcinoma

πŸ“Œ Rationale: Bartholin’s gland cysts are common in younger women, but in postmenopausal women (>60 years old), a mass in this area should raise suspicion for Bartholin’s gland carcinoma. (Document Reference: Bartholin’s Gland Carcinoma - β€œAny Bartholin’s gland enlargement in postmenopausal women should be considered malignant until proven otherwise.”)

How well did you know this?
1
Not at all
2
3
4
5
Perfectly
27
Q
  1. Identify the lesion: A soft, non-tender cystic mass near the vaginal opening.
A

Bartholin’s Gland Cyst

πŸ“Œ Rationale: Bartholin’s gland cysts are fluid-filled masses located near the vaginal opening. They are usually painless unless infected. (Document Reference: Bartholin’s Gland Cyst - β€œPainless unless infected.”)

How well did you know this?
1
Not at all
2
3
4
5
Perfectly
28
Q

Identify the raised pigmented lesion on the vulva.

A

Melanoma

πŸ“Œ Rationale: Vulvar melanoma presents as a darkly pigmented, raised lesion, often in elderly patients. It has a poor prognosis due to its aggressive nature. (Document Reference: Melanoma - β€œRaised pigmented lesion, poor prognosis.”)

How well did you know this?
1
Not at all
2
3
4
5
Perfectly
29
Q
  1. Vulvar cancer with lymph node involvement is classified as what FIGO stage?
    A. Stage II
    B. Stage IIIA
    C. Stage IIIC
    D. Stage IV
A

C. Stage IIIC

πŸ“Œ Rationale: Stage IIIC vulvar cancer indicates involvement of lymph nodes, regardless of tumor size. (Document Reference: Vulvar Cancer Staging - β€œIIIC = Lymph node involvement.”)

How well did you know this?
1
Not at all
2
3
4
5
Perfectly
30
Q
  1. Which of the following is a method to prevent HPV-related infections?
    A. Delaying sexual debut
    B. Hormonal contraceptives
    C. Avoiding alcohol
    D. High parity
A

A. Delaying sexual debut

πŸ“Œ Rationale: Delaying first sexual intercourse reduces the risk of early HPV exposure, lowering the chance of developing HPV-related cervical and vaginal neoplasms. (Document Reference: HPV Prevention - β€œDelaying sexual debut is a key primary prevention method.”)

How well did you know this?
1
Not at all
2
3
4
5
Perfectly
31
Q
  1. What is the treatment approach for early-stage vulvar cancer?
    A. Radical Local Excision
    B. Vulvectomy
    C. A or B
    D. Chemotherapy and Radiation
A

C. A or B (Radical Local Excision/Vulvectomy)

πŸ“Œ Rationale: Early-stage vulvar cancer is treated with surgical excision (Radical Local Excision or Vulvectomy). Chemotherapy and radiation are usually reserved for advanced or inoperable cases. (Document Reference: Vulvar Cancer Treatment - β€œEarly-stage disease managed surgically.”)

How well did you know this?
1
Not at all
2
3
4
5
Perfectly
32
Q
  1. Which of the following is NOT a method of preventing HPV-related infections?
    A. HPV vaccination
    B. Barrier contraceptive use
    C. Regular screening
    D. Bilateral Tubal Ligation (BTL)
A

D. Bilateral Tubal Ligation (BTL)

πŸ“Œ Rationale: BTL prevents pregnancy but has no effect on HPV infection risk. HPV prevention is achieved through vaccination, safe sexual practices, and screening. (Document Reference: HPV Prevention - β€œBTL prevents pregnancy, not HPV infections.”)

How well did you know this?
1
Not at all
2
3
4
5
Perfectly
33
Q
  1. Treatment for early-stage vulvar cancer consists of:
    A. Radical Local Excision
    B. Vulvectomy
    C. A and B
    D. Chemotherapy and Radiation
A

C. A and B (Radical Local Excision and Vulvectomy)

πŸ“Œ Rationale: Surgical excision is the standard treatment for early-stage vulvar cancer. Advanced cases may require chemotherapy and radiation. (Document Reference: Vulvar Cancer Treatment - β€œRadical Local Excision and Vulvectomy are standard approaches.”)

How well did you know this?
1
Not at all
2
3
4
5
Perfectly
34
Q
  1. Ways to prevent HPV-related infections, EXCEPT:
    A. Abstinence
    B. Pap smear
    C. HPV vaccine
    D. Tubal Ligation
A

D. Tubal Ligation

πŸ“Œ Rationale: Tubal ligation (BTL) is a contraceptive method that prevents pregnancy but does not protect against HPV infection. The best ways to prevent HPV-related infections include:

Abstinence (eliminates exposure to HPV)
HPV Vaccination (protects against high-risk HPV strains)
Regular Pap Smear Screening (detects early pre-cancerous changes but does not prevent HPV itself)

How well did you know this?
1
Not at all
2
3
4
5
Perfectly
35
Q
  1. A 48-year-old woman presents with post-coital vaginal bleeding and is found to have a cervical exophytic mass. A biopsy confirms cervical cancer, and molecular analysis of the specimen is performed. Which of the following HPV subtypes is most likely to be found?
    A. HPV 6 & 11
    B. HPV 16 & 18
    C. HPV 31 & 33
    D. HPV 45 & 52
A

B. HPV 16 & 18

πŸ“Œ Rationale: HPV 16 and 18 are the most common high-risk HPV types associated with cervical cancer, responsible for approximately 70% of cases worldwide. (Document Reference: Pathogenesis - HPV 16 & 18 are the most oncogenic.)

How well did you know this?
1
Not at all
2
3
4
5
Perfectly
36
Q
  1. The two most common high-risk HPV types causing cervical cancer in the Philippines are?
    A. HPV 6 & 11
    B. HPV 31 & 33
    C. HPV 16 & 18
    D. HPV 45 & 52
A

C. HPV 16 & 18

πŸ“Œ Rationale: HPV 16 and 18 cause approximately 70% of all cervical cancer cases worldwide, including in the Philippines. (Source: HPV Centre & WHO)

How well did you know this?
1
Not at all
2
3
4
5
Perfectly
37
Q
  1. A small percentage of women infected with HPV will develop cervical cancer in how many years?
    A. 5-10 years
    B. 10-15 years
    C. 15-20 years
    D. 25-30 years
A

C. 15-20 years

πŸ“Œ Rationale: HPV infection can persist and lead to cervical cancer, but it typically takes 15-20 years for cancer to develop in immunocompetent individuals. (Document Reference: WHO Fact Sheet)

How well did you know this?
1
Not at all
2
3
4
5
Perfectly
38
Q
  1. What is the most common symptom of early-stage cervical cancer?
    A. Pelvic pain
    B. Post-coital bleeding
    C. Abnormal vaginal discharge
    D. None
A

D. None

πŸ“Œ Rationale: Early-stage cervical cancer is often asymptomatic, which is why routine screening is crucial. Symptoms such as vaginal bleeding or discharge usually occur in later stages. (Document Reference: Clinical Manifestation)

How well did you know this?
1
Not at all
2
3
4
5
Perfectly
39
Q
  1. Which of the following statements is TRUE about cervical cancer?
    A. Cervical cancer is the most common malignancy in women
    B. Cervical cancer is the 3rd most common female cancer worldwide
    C. Cervical cancer only occurs in women with HPV
    D. Cervical cancer cannot be prevented
A

B. Cervical cancer is the 3rd most common female cancer worldwide

πŸ“Œ Rationale: Cervical cancer remains the third most common malignancy in women worldwide and is the most common gynecologic cancer in low-income countries. (Document Reference: Epidemiology)

How well did you know this?
1
Not at all
2
3
4
5
Perfectly
40
Q
  1. Which of the following is a risk factor for cervical cancer?
    A. Smoking
    B. Regular Pap smears
    C. Nulliparity
    D. Late age at first intercourse
A

A. Smoking

πŸ“Œ Rationale: Smoking increases the risk of cervical cancer by impairing immune response and promoting persistent HPV infection. (Document Reference: Risk Factors Table)

How well did you know this?
1
Not at all
2
3
4
5
Perfectly
41
Q
  1. A 28-year-old woman consults for cervical cancer screening. What is the recommended test?
    A. HPV test alone
    B. Pap smear alone
    C. Pap smear + HPV co-testing
    D. No screening needed
A

B. Pap smear alone

πŸ“Œ Rationale: Women aged 21-29 should have a Pap smear every 3 years. HPV testing alone is an option for women 25-29, but Pap smear is preferred. (Document Reference: Screening Guidelines)

How well did you know this?
1
Not at all
2
3
4
5
Perfectly
42
Q
  1. A 38-year-old woman (G2P2) has a Pap smear result showing HSIL. What is the next best step?
    A. Repeat Pap smear in 6 months
    B. HPV testing
    C. Colposcopy with possible cervical biopsy
    D. Immediate hysterectomy
A

C. Colposcopy with possible cervical biopsy

πŸ“Œ Rationale: HSIL requires further evaluation with colposcopy and potential biopsy to rule out invasive cancer. (Document Reference: HSIL Management)

How well did you know this?
1
Not at all
2
3
4
5
Perfectly
43
Q
  1. A cervical biopsy shows atypical cells and koilocytes occupying the lower 1/3 of the epithelium. What is the diagnosis?
    A. CIN 1
    B. CIN 2
    C. CIN 3
    D. Squamous cell carcinoma
A

A. CIN 1

πŸ“Œ Rationale: CIN 1 (LSIL) involves dysplasia confined to the lower 1/3 of the epithelium and is often associated with HPV infection. Koilocytes are hallmark cells of HPV infection. (Document Reference: CIN Classification)

How well did you know this?
1
Not at all
2
3
4
5
Perfectly
44
Q
  1. A 35-year-old woman with abnormal Pap smear underwent colposcopy-guided biopsy, which revealed HSIL. What is the recommended treatment?
    A. Watchful waiting
    B. Excisional procedure (LEEP or conization)
    C. HPV vaccination
    D. Repeat Pap smear in 6 months
A

B. Excisional procedure (LEEP or conization)

πŸ“Œ Rationale: HSIL is managed with an excisional procedure such as LEEP or cold knife conization to remove pre-cancerous cells and prevent progression to cervical cancer. (Document Reference: HSIL Treatment)

How well did you know this?
1
Not at all
2
3
4
5
Perfectly
45
Q
  1. What is the most common histologic type of cervical cancer?
    A. Adenocarcinoma
    B. Squamous cell carcinoma
    C. Small cell carcinoma
    D. Clear cell carcinoma
A

B. Squamous cell carcinoma

πŸ“Œ Rationale: Squamous cell carcinoma (SCC) accounts for approximately 70% of cervical cancer cases. (Document Reference: Histologic Types Table)

How well did you know this?
1
Not at all
2
3
4
5
Perfectly
46
Q
  1. A 39-year-old woman is diagnosed with cervical cancer that has spread to the right pelvic side wall. She has right hydronephrosis. What is the best treatment?
    A. Surgery alone
    B. Brachytherapy and external radiotherapy
    C. Chemotherapy alone
    D. HPV vaccination
A

B. Brachytherapy and external radiotherapy

πŸ“Œ Rationale: Cervical cancer with pelvic side wall involvement (Stage IIIB) is treated with radiotherapy (external + brachytherapy) combined with concurrent chemotherapy. (Document Reference: Cervical Cancer Treatment)

How well did you know this?
1
Not at all
2
3
4
5
Perfectly
47
Q
  1. Which of the following is a risk factor for cervical cancer?
    A. Early age of coitus
    B. Low parity
    C. Late menopause
    D. Lack of HPV vaccination
A

A. Early age of coitus

πŸ“Œ Rationale: Early sexual activity increases HPV exposure risk, leading to a higher likelihood of cervical dysplasia and cancer. (Document Reference: Risk Factors Table)

How well did you know this?
1
Not at all
2
3
4
5
Perfectly
48
Q
  1. A woman underwent total abdominal hysterectomy (TAH) for CIN 3. She asks if she still needs Pap smears. What is the correct advice?
    A. She should continue to have Pap smears of the vaginal cuff
    B. She no longer needs screening
    C. HPV vaccination will prevent recurrence
    D. Pap smear is only needed if symptoms appear
A

A. She should continue to have Pap smears of the vaginal cuff

πŸ“Œ Rationale: Women who undergo hysterectomy for high-grade CIN (CIN 2/3) should continue vaginal Pap smear screening since they remain at risk for vaginal cancer. (Document Reference: Post-Hysterectomy Screening)

How well did you know this?
1
Not at all
2
3
4
5
Perfectly
49
Q
  1. Vulvar cancer with involvement of lymph nodes is classified as which FIGO stage?
    A. Stage IIB
    B. Stage IIIA
    C. Stage IIIC
    D. Stage IVA
A

C. Stage IIIC

πŸ“Œ Rationale: Stage IIIC vulvar cancer indicates lymph node involvement. If distant metastases are present, it would be Stage IV. (Document Reference: Vulvar Cancer Staging - β€œIIIC = Lymph node involvement.”)

How well did you know this?
1
Not at all
2
3
4
5
Perfectly
50
Q
  1. A 53-year-old nulligravid woman underwent surgery for ovarian cancer. Her history revealed an untreated ovarian endometrioma diagnosed 10 years ago. What is the most likely histologic type of ovarian cancer?
    A. Serous carcinoma
    B. Mucinous carcinoma
    C. Clear cell adenocarcinoma
    D. Dysgerminoma
A

C. Clear cell adenocarcinoma

πŸ“Œ Rationale: Clear cell carcinoma is strongly associated with a history of endometriosis or ovarian endometriomas. (Document Reference: Ovarian Cancer Risk Factors - β€œEndometriosis increases risk for clear cell adenocarcinoma.”)

How well did you know this?
1
Not at all
2
3
4
5
Perfectly
51
Q
  1. A 46-year-old woman (G3P3) underwent total hysterectomy with bilateral salpingectomy for an ovarian mass. She had abdominal enlargement and pleural effusion. Histopathology confirmed a benign tumor, with no malignant cells in ascitic and pleural fluid. What is the most likely diagnosis?
    A. Fibroma
    B. Thecoma
    C. Granulosa cell tumor
    D. Dysgerminoma
A

A. Fibroma

πŸ“Œ Rationale: Fibroma is a benign solid ovarian tumor associated with ascites and pleural effusion, known as Meigs syndrome. (Document Reference: Ovarian Tumors - β€œFibroma with ascites and pleural effusion = Meigs syndrome.”)

How well did you know this?
1
Not at all
2
3
4
5
Perfectly
52
Q
  1. What is the most common type of germ cell tumor characterized by its ability to produce adult tissues like teeth and hair?
    A. Dysgerminoma
    B. Yolk sac tumor
    C. Mature cystic teratoma (Dermoid cyst)
    D. Immature teratoma
A

C. Mature cystic teratoma (Dermoid cyst)

πŸ“Œ Rationale: Mature cystic teratomas (dermoid cysts) are the most common germ cell tumors and contain fully differentiated tissues, such as hair, teeth, and sebaceous material. (Document Reference: Germ Cell Tumors - β€œMature teratomas contain well-differentiated tissues.”)

How well did you know this?
1
Not at all
2
3
4
5
Perfectly
53
Q
  1. A 28-year-old nulligravid woman presents with urinary frequency. Ultrasound shows a 9 cm benign tumor, likely a mature cystic teratoma. What is the recommended treatment?
    A. Cystectomy
    B. Hysterectomy
    C. Salpingo-oophorectomy
    D. Observation
A

A. Cystectomy

πŸ“Œ Rationale: Cystectomy (removal of the cyst while preserving ovarian tissue) is the preferred treatment for young women with benign ovarian tumors to preserve fertility. (Document Reference: Management of Ovarian Tumors)

How well did you know this?
1
Not at all
2
3
4
5
Perfectly
54
Q
  1. Which histologic type accounts for 5% of ovarian cancers and is characterized by hobnail cells protruding into glandular lumens?
    A. Dysgerminoma
    B. Immature teratoma
    C. Clear cell carcinoma
    D. Mucinous carcinoma
A

C. Clear cell carcinoma

πŸ“Œ Rationale: Clear cell carcinoma comprises 5% of ovarian cancers and is characterized by hobnail cells. It is commonly associated with endometriosis. (Document Reference: Ovarian Cancer Histology)

How well did you know this?
1
Not at all
2
3
4
5
Perfectly
55
Q
  1. What is the recommended adjuvant therapy for patients with frankly malignant ovarian cancer (Stage IC-IVB)?
    A. Radiation therapy
    B. Chemotherapy
    C. Hormonal therapy
    D. Surgery only
A

B. Chemotherapy

πŸ“Œ Rationale: Patients with Stage IC-IVB ovarian cancer require adjuvant chemotherapy, typically with platinum-based regimens (e.g., carboplatin + paclitaxel). (Document Reference: Ovarian Cancer Treatment - β€œChemotherapy is recommended for Stage IC-IVB.”)

How well did you know this?
1
Not at all
2
3
4
5
Perfectly
56
Q
  1. A 15-year-old girl presents with abdominal enlargement. Ultrasound reveals a solid ovarian mass. What tumor markers should be requested?
    A. CA-125, hCG, AFP, and LDH
    B. CA-19-9 and CEA
    C. BRCA1 and BRCA2
    D. Estrogen and progesterone
A

A. CA-125, hCG, AFP, and LDH

πŸ“Œ Rationale: These tumor markers are used to evaluate germ cell tumors:

CA-125 (epithelial ovarian tumors)
hCG (choriocarcinoma)
AFP (yolk sac tumors)
LDH (dysgerminoma)
(Document Reference: Tumor Markers in Ovarian Cancer)

How well did you know this?
1
Not at all
2
3
4
5
Perfectly
57
Q
  1. What is the most frequently found immature tissue in immature teratomas?
    A. Immature neuroepithelium
    B. Immature cartilage
    C. Immature muscle
    D. Immature squamous cells
A

A. Immature neuroepithelium

πŸ“Œ Rationale: Immature teratomas contain embryonal tissue, most commonly immature neuroepithelium, which is associated with aggressive behavior. (Document Reference: Germ Cell Tumors)

How well did you know this?
1
Not at all
2
3
4
5
Perfectly
58
Q
  1. What is the most common histologic type of fallopian tube cancer?
    A. Serous adenocarcinoma
    B. Mucinous carcinoma
    C. Clear cell carcinoma
    D. Squamous cell carcinoma
A

A. Serous adenocarcinoma

πŸ“Œ Rationale: Serous adenocarcinoma is the most common histologic type of fallopian tube carcinoma, resembling high-grade serous ovarian carcinoma. (Document Reference: Fallopian Tube Cancer Histology)

How well did you know this?
1
Not at all
2
3
4
5
Perfectly
59
Q
  1. The field effect in peritoneal carcinoma suggests that:
    A. Peritoneal cancer originates from mesothelial cells
    B. The epithelium lining the abdominal cavity and germinal epithelium respond similarly to carcinogenic stimuli
    C. BRCA mutations are not relevant to peritoneal carcinoma
    D. Peritoneal carcinoma arises exclusively from ovarian cancer
A

B. The epithelium lining the abdominal cavity and germinal epithelium respond similarly to carcinogenic stimuli

πŸ“Œ Rationale: The field effect theory states that peritoneal and ovarian cancers share a common carcinogenic response, explaining why some peritoneal carcinomas arise independently of ovarian cancer. (Document Reference: Peritoneal Carcinoma - β€œField Effect Theory”)

How well did you know this?
1
Not at all
2
3
4
5
Perfectly
60
Q
  1. Which statement is TRUE about ovarian tumor rupture during surgery?
    A. Patients whose tumor structure remains intact during surgery have better survival rates than those with pre-surgical rupture
    B. Tumor rupture has no effect on prognosis
    C. Pre-surgical tumor rupture improves survival
    D. Tumor rupture is an indication for chemotherapy
A

A. Patients whose tumor structure remains intact during surgery have better survival rates than those with pre-surgical rupture

πŸ“Œ Rationale: Ovarian tumor rupture before surgery leads to intraperitoneal spread, worsening prognosis. Intraoperative rupture is also a risk but has a better outcome than preoperative rupture. (Document Reference: Ovarian Cancer Prognosis)

How well did you know this?
1
Not at all
2
3
4
5
Perfectly
61
Q
  1. In a young patient undergoing surgery for epithelial ovarian cancer, the uterus and contralateral ovary may be preserved under which condition?
    A. The tumor is bilateral
    B. The tumor is poorly differentiated
    C. The tumor is confined to one ovary, with no capsular invasion, and is well-differentiated
    D. There is lymph node involvement
A

C. The tumor is confined to one ovary, with no capsular invasion, and is well-differentiated

πŸ“Œ Rationale: Fertility-preserving surgery is an option in early-stage ovarian cancer if the tumor is unilateral, well-differentiated, and has no capsular breach or metastasis. (Document Reference: Fertility-Preserving Surgery in Ovarian Cancer)

How well did you know this?
1
Not at all
2
3
4
5
Perfectly
62
Q
  1. A 45-year-old woman with a history of breast cancer presents for evaluation. What is the next step in management?
    A. Perform an endometrial biopsy only if the endometrial thickness is >4 mm
    B. Observe unless she has vaginal bleeding
    C. If on tamoxifen and experiencing irregular bleeding, perform an endometrial biopsy
    D. Routine transvaginal ultrasound is sufficient
A

C. If on tamoxifen and experiencing irregular bleeding, perform an endometrial biopsy

πŸ”Ž Rationale:

Tamoxifen increases the risk of endometrial hyperplasia and cancer but does not warrant routine screening unless the patient has abnormal bleeding.
Endometrial biopsy is only done if the patient has irregular bleeding, regardless of ultrasound findings.
β€œWe only do biopsy kapag dinudugo. Pag walang dugo, walang biopsy kahit makapal ang endometrium sa ultrasound.”

How well did you know this?
1
Not at all
2
3
4
5
Perfectly
63
Q
  1. An autosomal dominant syndrome characterized by germline defective DNA repair is associated with an increased risk of endometrial cancer. What should be advised for this patient?
    A. Endometrial biopsy every 5 years
    B. Routine pelvic examination only
    C. Annual transvaginal ultrasound
    D. OCPs as preventive therapy
A

C. Annual transvaginal ultrasound

πŸ”Ž Rationale:

Lynch Syndrome (Hereditary Nonpolyposis Colorectal Cancer - HNPCC) increases the risk of endometrial and colon cancer (40-60% lifetime risk).
Recommended screening: Annual endometrial biopsy and transvaginal ultrasound for early detection.

How well did you know this?
1
Not at all
2
3
4
5
Perfectly
64
Q
  1. Which of the following is NOT a risk factor for endometrial cancer?
    A. Obesity
    B. Late menopause (>55 years old)
    C. Anorexia
    D. Tamoxifen use
A

C. Anorexia

πŸ”Ž Rationale:

Risk factors for endometrial cancer include:
Unopposed estrogen stimulation (e.g., obesity, PCOS, tamoxifen)
More years of menstruation (early menarche, late menopause)
Nulliparity (never being pregnant)
Older age and hormone therapy
Lynch syndrome (HNPCC)
Anorexia does not increase riskβ€”it reduces estrogen levels, which lowers the risk of endometrial hyperplasia and cancer.

How well did you know this?
1
Not at all
2
3
4
5
Perfectly
65
Q
  1. A patient on tamoxifen asks about the risk of endometrial cancer. What should you tell her?
    A. Routine endometrial biopsy is necessary every 6 months
    B. Transvaginal ultrasound should be done annually
    C. Routine lab work including CBC, calcium, and liver function tests should be done
    D. Endometrial biopsy is only needed if endometrial thickness exceeds 5 mm
A

C. Routine lab work including CBC, calcium, and liver function tests should be done

πŸ”Ž Rationale:

Tamoxifen acts as an anti-estrogen in the breast but as an estrogen agonist in the uterus, increasing the risk of endometrial hyperplasia and cancer.
Routine endometrial biopsy is NOT needed unless the patient has vaginal bleeding.
Patients on tamoxifen should have routine lab tests (CBC, liver function, calcium levels).

How well did you know this?
1
Not at all
2
3
4
5
Perfectly
66
Q
  1. What is the treatment of choice for vulvar basal cell carcinoma?
    A. Radiation therapy
    B. Wide local excision
    C. Radical vulvectomy
    D. Chemotherapy
A

B. Wide local excision

πŸ”Ž Rationale:

Basal cell carcinoma of the vulva is treated with wide local excision, as it is a slow-growing tumor with low metastatic potential.
Lymph node dissection is not required unless deeper invasion is suspected.

How well did you know this?
1
Not at all
2
3
4
5
Perfectly
67
Q
  1. Which of the following statements about endometrial cancer is TRUE?
    A. It is the most common genital tract malignancy in the Philippines
    B. It is the third most common genital tract malignancy in the Philippines
    C. It is always associated with HPV
    D. It has a poorer prognosis than cervical cancer
A

B. It is the third most common genital tract malignancy in the Philippines

πŸ”Ž Rationale:

Endometrial cancer ranks 3rd among genital tract malignancies in the Philippines, after cervical cancer and ovarian cancer.

How well did you know this?
1
Not at all
2
3
4
5
Perfectly
68
Q
  1. Conization is indicated for the diagnosis of women with HSIL based on a Pap smear under which condition?
    A. The limits of the lesion cannot be visualized with colposcopy
    B. The squamo-columnar junction is not visualized
    C. Endocervical curettage histologic findings are positive for CIN 2 or 3
    D. All of the above
A

D. All of the above

πŸ”Ž Rationale:

Conization is recommended when the lesion cannot be fully assessed via colposcopy, when the squamo-columnar junction is not visualized, or if endocervical curettage is positive for CIN 2 or 3.

How well did you know this?
1
Not at all
2
3
4
5
Perfectly
69
Q
  1. Endometrial carcinoma is generally regarded as a hormone-dependent neoplasm for which of the following reasons?
    A. It arises in a milieu of unopposed estrogen
    B. It is preceded by endometrial hyperplasia
    C. It responds to progestin therapy
    D. All of the above
A

D. All of the above

πŸ”Ž Rationale:

Endometrial carcinoma is an estrogen-driven malignancy.
It develops in settings of unopposed estrogen stimulation (e.g., PCOS, obesity, late menopause, tamoxifen use).
It often follows endometrial hyperplasia and responds to progestin therapy in some cases.

How well did you know this?
1
Not at all
2
3
4
5
Perfectly
70
Q
  1. Which of the following is synonymous with Endometrial Intraepithelial Neoplasia (EIN)?
    A. Simple hyperplasia
    B. Complex atypical hyperplasia
    C. Clear cell carcinoma
    D. Papillary serous carcinoma
A

B. Complex atypical hyperplasia

πŸ”Ž Rationale:

Complex atypical hyperplasia is now referred to as Endometrial Intraepithelial Neoplasia (EIN) in the updated WHO classification.
EIN is a premalignant lesion with a high risk (29-42%) of progression to endometrial carcinoma.

How well did you know this?
1
Not at all
2
3
4
5
Perfectly
71
Q
  1. A 40-year-old woman with vaginal bleeding is diagnosed with complex atypical hyperplasia. Which of the following is true?
    A. Endometrial biopsy has a 10% chance of revealing endometrial cancer
    B. Endometrial biopsy has a 42% chance of detecting concurrent endometrial cancer
    C. Complex hyperplasia has no risk of malignancy
    D. Simple hyperplasia is highly malignant
A

B. Endometrial biopsy has a 42% chance of detecting concurrent endometrial cancer

πŸ”Ž Rationale:

Complex atypical hyperplasia (EIN) is highly premalignant.
42% of women with complex atypical hyperplasia already have endometrial carcinoma at the time of hysterectomy.
Management:
Progestin therapy (if fertility is desired).
Hysterectomy (definitive treatment, especially in postmenopausal women).

How well did you know this?
1
Not at all
2
3
4
5
Perfectly
72
Q
  1. Complex hyperplasia with atypia is associated with which of the following?
    A. It has no risk of malignancy
    B. 10% of cases progress to endometrial carcinoma
    C. 42% of women with complex atypical hyperplasia have endometrial carcinoma
    D. It cannot be diagnosed with endometrial biopsy
A

C. 42% of women with complex atypical hyperplasia have endometrial carcinoma

πŸ”Ž Rationale:

Complex hyperplasia with atypia (Endometrial Intraepithelial Neoplasia - EIN) is highly premalignant.
42% of cases already have endometrial carcinoma at the time of hysterectomy.
D&C or endometrial sampling may miss areas of carcinoma mixed with hyperplasia.

How well did you know this?
1
Not at all
2
3
4
5
Perfectly
73
Q
  1. Which of the following is NOT a diagnostic test for endometrial cancer?
    A. Endometrial biopsy
    B. Transvaginal ultrasound
    C. Dilatation & curettage (D&C)
    D. Keyes-punch biopsy
A

D. Keyes-punch biopsy

πŸ”Ž Rationale:

Keyes-punch biopsy is used for diagnosing vulvar cancer, NOT endometrial cancer.
Endometrial biopsy and D&C are the preferred diagnostic tools for endometrial cancer.

How well did you know this?
1
Not at all
2
3
4
5
Perfectly
74
Q
  1. A 50-year-old postmenopausal woman has had intermittent vaginal bleeding for 7 months. Ultrasound reveals a 3.8 mm endometrial mass. What is the next step?
    A. Observation
    B. Endometrial biopsy
    C. Hormone therapy
    D. Repeat ultrasound in 6 months
A

B. Endometrial biopsy

πŸ”Ž Rationale:

Postmenopausal bleeding is always abnormal and must be evaluated.
Dr. Malig states that biopsy should be done immediately when there is bleeding, regardless of endometrial thickness.

How well did you know this?
1
Not at all
2
3
4
5
Perfectly
75
Q
  1. A patient presents with irregular vaginal bleeding and is diagnosed with hyperplasia without atypia. What is the recommended management?
    A. Hysterectomy
    B. Chemotherapy
    C. Progestin therapy
    D. Radiation therapy
A

C. Progestin therapy

πŸ”Ž Rationale:

Hyperplasia without atypia is managed with progestin therapy to thin the endometrial lining.
Medroxyprogesterone acetate or oral contraceptive pills (OCPs) can be used.

How well did you know this?
1
Not at all
2
3
4
5
Perfectly
76
Q
  1. A 29-year-old obese, nulligravid woman is diagnosed with complex atypical hyperplasia. What is the best treatment option?
    A. Observation
    B. High-dose progestin with periodic endometrial sampling
    C. Immediate hysterectomy
    D. Radiation therapy
A

B. High-dose progestin with periodic endometrial sampling

πŸ”Ž Rationale:

Obesity is a risk factor for endometrial hyperplasia due to increased estrogen production.
In young patients who desire fertility, high-dose progestins (Megestrol Acetate 160 mg/day) are used with periodic biopsies.
If hyperplasia persists or progresses, hysterectomy is recommended.

How well did you know this?
1
Not at all
2
3
4
5
Perfectly
77
Q
  1. A 55-year-old woman presents with vaginal bleeding. A biopsy confirms the diagnosis of endometrial adenocarcinoma. Which statement is TRUE?
    A. It is the most common genital malignancy
    B. It primarily occurs in premenopausal women
    C. It is associated with poor prognosis due to early metastasis
    D. High tumor grade, advanced stage, and lymphovascular invasion worsen prognosis
A

D. High tumor grade, advanced stage, and lymphovascular invasion worsen prognosis

πŸ”Ž Rationale:

Endometrial adenocarcinoma is the 3rd most common genital tract malignancy.
It occurs primarily in postmenopausal women.
Poor prognostic factors include:
High tumor grade
Advanced surgical stage
Lymphovascular space invasion

How well did you know this?
1
Not at all
2
3
4
5
Perfectly
78
Q
  1. Which histologic type of endometrial carcinoma has the WORST prognosis?
    A. Endometrioid adenocarcinoma
    B. Mucinous carcinoma
    C. Papillary serous carcinoma
    D. Clear cell carcinoma
A

D. Clear cell carcinoma

πŸ”Ž Rationale:

Clear cell carcinoma is highly aggressive with a poor prognosis.
It has a high likelihood of vascular invasion and distant metastasis.

How well did you know this?
1
Not at all
2
3
4
5
Perfectly
79
Q
  1. A solid tumor is found on the endometrium with involvement of the cervical stroma. What is the FIGO stage?
    A. Stage IA
    B. Stage IB
    C. Stage II
    D. Stage IIIA
A

C. Stage II

πŸ”Ž Rationale:

Stage II endometrial cancer involves the cervical stroma but remains confined to the uterus.
It requires hysterectomy and possibly adjuvant therapy.

How well did you know this?
1
Not at all
2
3
4
5
Perfectly
80
Q
  1. A 49-year-old woman undergoes surgery for endometrial cancer. A 2x3 cm tumor is found in the endometrium with NO myometrial invasion. What is the FIGO stage?
    A. Stage IA
    B. Stage IB
    C. Stage II
    D. Stage III
A

A. Stage IA

πŸ”Ž Rationale:

Stage IA: Tumor is confined to the endometrium or has <50% myometrial invasion.
Stage IB: Tumor invades >50% of the myometrium.
Since there is no invasion, this is Stage IA.

How well did you know this?
1
Not at all
2
3
4
5
Perfectly
81
Q
  1. What is the best treatment option for a postmenopausal woman with confirmed Stage IA endometrial carcinoma?
    A. Total abdominal hysterectomy with bilateral salpingo-oophorectomy (TAHBSO)
    B. Chemotherapy
    C. Radiation therapy only
    D. Hormonal therapy with progestins
A

A. Total abdominal hysterectomy with bilateral salpingo-oophorectomy (TAHBSO)

πŸ”Ž Rationale:

Surgery (TAHBSO) is the standard treatment for early-stage endometrial cancer (Stage IA).
Adjuvant therapy (radiation or chemotherapy) is considered for high-risk cases (e.g., high-grade tumors).

How well did you know this?
1
Not at all
2
3
4
5
Perfectly
82
Q
  1. What is the estimated 5-year survival rate for a patient with Stage IA endometrial carcinoma?
    A. 50%
    B. 70%
    C. 90%
    D. 40%
A

C. 90%

πŸ”Ž Rationale:

Stage IA endometrial cancer has an excellent prognosis with a 5-year survival rate of 90% if treated appropriately with surgery.
πŸ“Œ Reference: FIGO Staging and 5-year survival rates for endometrial carcinoma.

How well did you know this?
1
Not at all
2
3
4
5
Perfectly
83
Q
  1. A patient’s tumor is well-differentiated with less than 5% solid components. What is the tumor grade?
    A. Grade 1
    B. Grade 2
    C. Grade 3
    D. High grade
A

A. Grade 1

πŸ”Ž Rationale:

Tumor grading is based on the percentage of solid components:
Grade 1 = Well-differentiated, <5% solid components
Grade 2 = Moderately differentiated, 6-50% solid components
Grade 3 = Poorly differentiated, >50% solid components

How well did you know this?
1
Not at all
2
3
4
5
Perfectly
84
Q
  1. Which of the following is considered a poor prognostic factor EXCEPT?
    A. Papillary serous carcinoma
    B. Clear cell carcinoma
    C. Poorly differentiated carcinoma
    D. Typical endometrioid adenocarcinoma
A

D. Typical endometrioid adenocarcinoma

πŸ”Ž Rationale:

Endometrioid adenocarcinoma has the best prognosis.
Papillary serous and clear cell carcinomas are highly aggressive with poor outcomes.
Poor differentiation (Grade 3) also correlates with poor prognosis.
πŸ“Œ Reference: Histologic subtypes and prognostic factors in endometrial carcinoma.

How well did you know this?
1
Not at all
2
3
4
5
Perfectly
85
Q
  1. A patient with endometrial cancer presents with enlarged inguinal lymph nodes. What is the most likely route of spread?
    A. Bloodstream
    B. Round ligament
    C. Fallopian tubes
    D. Peritoneal fluid
A

B. Round ligament

πŸ”Ž Rationale:

Endometrial cancer spreads to the inguinal lymph nodes via the round ligament.
Other routes of spread include:
Pelvic & para-aortic lymph nodes
Peritoneal fluid (direct spread)
Fallopian tubes (retrograde spread)

How well did you know this?
1
Not at all
2
3
4
5
Perfectly
86
Q
  1. What is the primary treatment for endometrial carcinoma?
    A. Chemotherapy
    B. Radiation therapy
    C. Concurrent chemoradiation
    D. Surgery
A

D. Surgery

πŸ”Ž Rationale:

Total abdominal hysterectomy with bilateral salpingo-oophorectomy (TAHBSO) is the standard treatment for endometrial cancer.
Adjuvant therapy (radiation or chemotherapy) is added based on risk factors.

How well did you know this?
1
Not at all
2
3
4
5
Perfectly
87
Q

What is the most important predictor of survival in endometrial sarcoma?
A. Tumor grading
B. Histologic type
C. Stage
D. Age of the patient

A

C. Stage

πŸ”Ž Rationale:

Stage is the most important predictor of survival in endometrial sarcoma.
Higher stages correlate with poorer outcomes.
Other factors like tumor grade and histologic type also influence prognosis but are secondary to stage.

How well did you know this?
1
Not at all
2
3
4
5
Perfectly
88
Q
  1. A 48-year-old woman with endometrial stromal sarcoma (ESS) is found to have estrogen and progesterone receptors. What does this indicate?
    A. Resistance to hormone therapy
    B. Tumor is aggressive and unresponsive to treatment
    C. Tumor may respond to hormonal therapy
    D. No clinical significance
A

C. Tumor may respond to hormonal therapy

πŸ”Ž Rationale:

Endometrial stromal sarcomas often express estrogen and progesterone receptors.
Hormonal therapy (progestins, aromatase inhibitors) can be used to slow tumor progression.

How well did you know this?
1
Not at all
2
3
4
5
Perfectly
89
Q
  1. A 41-year-old woman undergoes total hysterectomy for a uterine mass. Histopathology shows >5 mitoses per 10 HPF. What is the most likely diagnosis?
    A. Endometrial carcinoma
    B. Endometrial stromal sarcoma
    C. Leiomyosarcoma
    D. Adenomyosis
A

C. Leiomyosarcoma

πŸ”Ž Rationale:

Leiomyosarcoma is a rare but aggressive smooth muscle tumor of the uterus.
Histologic criteria include >5 mitoses per 10 high-power fields (HPF).

How well did you know this?
1
Not at all
2
3
4
5
Perfectly
90
Q
  1. What is the estimated lifetime risk for an individual to develop ovarian cancer?
    A. 0.5%
    B. 1.3%
    C. 5%
    D. 10%
A

B. 1.3%

πŸ”Ž Rationale:

The average woman’s lifetime risk of developing ovarian cancer is approximately 1.3%.
Risk increases with BRCA1/BRCA2 mutations or family history.

How well did you know this?
1
Not at all
2
3
4
5
Perfectly
91
Q
  1. An endometrial mass composed of both carcinomatous and sarcomatous elements is known as which of the following?
    A. Leiomyosarcoma
    B. Endometrial stromal sarcoma
    C. Mixed MΓΌllerian tumor
    D. Clear cell carcinoma
A

C. Mixed MΓΌllerian tumor

πŸ”Ž Rationale:

Malignant Mixed MΓΌllerian Tumor (MMMT) contains both epithelial (carcinomatous) and mesenchymal (sarcomatous) components.
It is highly aggressive with a poor prognosis.
πŸ“Œ Reference: Histology of mixed MΓΌllerian tumors.

92
Q
  1. A 48-year-old woman with Endometrial Stromal Sarcoma (ESS) is found to have estrogen and progesterone receptors. What is the clinical significance of this finding?
    A. The tumor is aggressive and unresponsive to treatment
    B. The tumor is resistant to hormonal therapy
    C. The tumor may respond to hormonal therapy
    D. No clinical significance
A

C. The tumor may respond to hormonal therapy

πŸ”Ž Rationale:

Endometrial Stromal Sarcoma (ESS) is often estrogen and progesterone receptor-positive.
This means the tumor may respond to hormonal therapy, such as:
Progestins (e.g., Medroxyprogesterone acetate)
Aromatase inhibitors (e.g., Letrozole, Anastrozole)
GnRH agonists (e.g., Leuprolide)

93
Q
  1. A 56-year-old woman presents with a 2 cm ulcerating lesion at the right labia majora, present for 5 months. A punch biopsy reveals squamous cell carcinoma. What is the most likely site of metastasis?
    A. Inguinal lymph nodes
    B. Para-aortic lymph nodes
    C. Pelvic lymph nodes
    D. Liver
    E. Lungs
A

A. Inguinal lymph nodes

πŸ”Ž Rationale:

Vulvar squamous cell carcinoma (SCC) primarily metastasizes to the inguinal lymph nodes.
Pelvic lymph node involvement occurs at later stages.
Distant metastasis to the lungs and liver is rare in early-stage disease.
πŸ“Œ Source: Gyne LE 5.02 – Premalignant & Malignant Lesions of the Uterus

94
Q
  1. A 60-year-old woman presents with postmenopausal bleeding and is diagnosed with endometrial cancer after endometrial sampling. Which of the following is a risk factor for endometrial carcinoma?
    A. Nulliparity
    B. Diabetes
    C. Early menopause
    D. HPV infection
    E. Smoking
A

B. Diabetes

πŸ”Ž Rationale:

Diabetes is an independent risk factor for endometrial cancer.
Other major risk factors include:
Obesity
Late menopause (>55 years old)
Unopposed estrogen therapy
PCOS
HPV and smoking are risk factors for cervical cancer, not endometrial cancer.

95
Q
  1. A 53-year-old woman with a history of stage 2 breast cancer has been taking tamoxifen for 3 years and complains of vaginal spotting. A transvaginal ultrasound shows thickened endometrium. What is the next best step?
    A. Observe, the transvaginal ultrasound might be a false positive
    B. Endometrial biopsy
    C. Repeat transvaginal ultrasound in 6 months
    D. Start progestin therapy
    E. Hysteroscopy
A

B. Endometrial biopsy

πŸ”Ž Rationale:

Tamoxifen is an estrogen agonist in the uterus, increasing the risk of endometrial hyperplasia and carcinoma.
A thickened endometrium on ultrasound + vaginal spotting warrants an endometrial biopsy to rule out malignancy.
Observation is inappropriate as persistent vaginal bleeding requires further investigation.

96
Q
  1. What autosomal dominant syndrome is characterized by germline defects in DNA repair genes and predisposes individuals to a 40-60% chance of developing endometrial and colon cancer?
    A. Lynch syndrome
    B. Li-Fraumeni syndrome
    C. BRCA1 mutation
    D. Cowden syndrome
    E. Peutz-Jeghers syndrome
A

A. Lynch syndrome

πŸ”Ž Rationale:

Lynch syndrome (Hereditary Nonpolyposis Colorectal Cancer - HNPCC) is caused by mutations in DNA mismatch repair genes (MLH1, MSH2, MSH6, PMS2).
It significantly increases the risk of colorectal and endometrial cancer (40-60% lifetime risk).

97
Q
  1. Which of the following should be advised to a patient diagnosed with Lynch syndrome?
    A. Annual transvaginal ultrasound
    B. Routine endometrial biopsy every 10 years
    C. CA-125 monitoring
    D. Avoidance of OCPs
    E. Pelvic MRI every 5 years
A

A. Annual transvaginal ultrasound

πŸ”Ž Rationale:

Women with Lynch syndrome should undergo annual transvaginal ultrasound and endometrial biopsy starting at age 35.
Prophylactic hysterectomy and bilateral salpingo-oophorectomy (TAHBSO) are recommended after childbearing.
πŸ“Œ Source: Gyne LE 5.02 and Feedback

98
Q
  1. Which of the following increases the risk of endometrial cancer?
    A. Menopause at age 55
    B. Multiparity
    C. Early menopause
    D. Use of combination OCPs
    E. Low BMI
A

A. Menopause at age 55

πŸ”Ž Rationale:

Late menopause (>55 years) increases estrogen exposure, raising the risk of endometrial cancer.
Protective factors include:
Early menopause
Multiparity (progesterone exposure during pregnancy)
Use of combination OCPs

99
Q
  1. Which condition is synonymous with Endometrial Intraepithelial Neoplasia (EIN)?
    A. Complex hyperplasia with atypia
    B. Simple hyperplasia without atypia
    C. Endometrial stromal sarcoma
    D. Clear cell carcinoma
    E. Papillary serous carcinoma
A

βœ… Answer: A. Complex hyperplasia with atypia

πŸ”Ž Rationale:

Complex hyperplasia with atypia is now classified as Endometrial Intraepithelial Neoplasia (EIN) in the updated WHO system.
EIN has a 29-42% risk of progressing to endometrial carcinoma.

100
Q
  1. A 40-year-old woman presents with vaginal bleeding and is diagnosed with complex atypical hyperplasia. Which of the following is true?
    A. Endometrial biopsy can have a 40% chance of detecting endometrial carcinoma
    B. Complex atypical hyperplasia always progresses to cancer
    C. Endometrial carcinoma is unlikely to develop in this patient
    D. The patient should be managed with observation only
    E. Complex atypical hyperplasia is considered benign
A

A. Endometrial biopsy can have a 40% chance of detecting endometrial carcinoma

πŸ”Ž Rationale:

Complex atypical hyperplasia (Endometrial Intraepithelial Neoplasia - EIN) is a premalignant condition.
Approximately 42% of cases of complex atypical hyperplasia are found to have endometrial carcinoma at the time of hysterectomy.
Management options include high-dose progestin therapy (for fertility preservation) or hysterectomy (definitive treatment).

101
Q
  1. Which of the following is NOT a diagnostic test for endometrial cancer?
    A. Keyes-punch biopsy
    B. Endometrial biopsy
    C. Transvaginal ultrasound
    D. Dilatation & curettage (D&C)
    E. Hysteroscopy
A

A. Keyes-punch biopsy

πŸ”Ž Rationale:

Keyes-punch biopsy is used for diagnosing vulvar cancer, NOT endometrial cancer.
Preferred diagnostic methods for endometrial cancer include:
Endometrial biopsy (gold standard for outpatient diagnosis)
Transvaginal ultrasound (for initial evaluation of endometrial thickness)
Dilatation & Curettage (D&C) for inconclusive biopsy results
Hysteroscopy for direct visualization and biopsy

102
Q
  1. A 50-year-old postmenopausal woman has had intermittent vaginal bleeding for several months. Ultrasound reveals a 3.8 mm endometrial mass. What is the next step in management?
    A. Biopsy
    B. Observe and repeat ultrasound in 6 months
    C. Prescribe progesterone therapy
    D. Perform a hysterectomy immediately
    E. Reassure the patient and advise lifestyle modification
A

A. Biopsy

πŸ”Ž Rationale:

Postmenopausal bleeding is always abnormal and requires further evaluation.
Endometrial biopsy should be performed regardless of endometrial thickness if there is persistent vaginal bleeding.
Dr. Malig states that biopsy should be done immediately when bleeding is present, even if the ultrasound shows a thin endometrium.

103
Q
  1. A 45-year-old G0 woman with a history of hypertension, diabetes, and PCOS presents with 2 weeks of vaginal bleeding. She has atrophic endometrial cells and was given estrogen-replacement therapy. Three months later, she experiences another episode of bleeding. What is the next best step?
    A. Assure that this is normal
    B. Start estrogen-progesterone therapy
    C. Perform an endometrial biopsy
    D. Increase the dose of estrogen therapy
A

C. Perform an endometrial biopsy

πŸ”Ž Rationale:

Postmenopausal bleeding after estrogen replacement therapy requires further evaluation.
Unopposed estrogen therapy increases the risk of endometrial hyperplasia and cancer.
PCOS is already a risk factor for endometrial carcinoma due to chronic anovulation and unopposed estrogen.
Endometrial biopsy is needed to rule out endometrial hyperplasia or malignancy.

104
Q
  1. A 29-year-old woman with hypertension and diabetes is diagnosed with Stage I endometrial carcinoma. What is the best therapeutic management?
    A. Chemotherapy
    B. Radiation therapy
    C. Progesterone therapy
    D. Surgery
A

D. Surgery

πŸ”Ž Rationale:

Stage I endometrial cancer is best managed with surgery (Total Abdominal Hysterectomy with Bilateral Salpingo-Oophorectomy - TAHBSO).
Adjuvant therapy (radiation or chemotherapy) is considered for high-risk Stage I cases.
Progesterone therapy is only used in fertility-sparing cases.

105
Q
  1. A 35-year-old woman is diagnosed with endometrial cancer. Which of the following conditions is most likely present?
    A. Polycystic Ovarian Syndrome (PCOS)
    B. HPV infection
    C. BRCA1 mutation
    D. Smoking history
A

A. Polycystic Ovarian Syndrome (PCOS)

πŸ”Ž Rationale:

PCOS is a major risk factor for endometrial cancer due to chronic anovulation and unopposed estrogen stimulation.
HPV infection is associated with cervical cancer, not endometrial cancer.
BRCA1 mutations increase the risk of ovarian and breast cancer, not primarily endometrial cancer.
Smoking is a risk factor for cervical cancer, but it has a paradoxically protective effect against endometrial cancer.

106
Q
  1. What is the primary treatment for endometrial cancer?
    A. Chemotherapy
    B. Radiation therapy
    C. Hormonal therapy
    D. Surgery
    E. Observation
A

D. Surgery

πŸ”Ž Rationale:

Surgical management (Total Abdominal Hysterectomy with Bilateral Salpingo-Oophorectomy - TAHBSO) is the gold standard for most cases of endometrial cancer.
Adjuvant therapy (radiation or chemotherapy) is considered based on the tumor grade, depth of invasion, and lymph node involvement.
Hormonal therapy (progestins) is used only for fertility-sparing management in select cases.

107
Q
  1. A 45-year-old G0 woman with a history of hypertension, diabetes, and PCOS presents with 2 weeks of vaginal bleeding. She has atrophic endometrial cells and was given estrogen-replacement therapy. Three months later, she experiences another episode of bleeding. What is the next best step?
    A. Assure that this is normal
    B. Start estrogen-progesterone therapy
    C. Perform an endometrial biopsy
    D. Increase the dose of estrogen therapy
A

C. Perform an endometrial biopsy

πŸ”Ž Rationale:

Postmenopausal bleeding after estrogen replacement therapy warrants further investigation.
PCOS, diabetes, and obesity are risk factors for endometrial hyperplasia and cancer.
Endometrial biopsy is needed to rule out endometrial carcinoma or hyperplasia.

108
Q
  1. A patient is diagnosed with Stage 4A endometrial cancer. What is the extent of disease at this stage?
    A. Tumor invades the bladder and/or bowel mucosa
    B. Tumor is confined to the uterus
    C. Tumor has spread to the para-aortic lymph nodes
    D. Tumor has metastasized to the liver and lungs
    E. Tumor has spread to the cervix only
A

A. Tumor invades the bladder and/or bowel mucosa

πŸ”Ž Rationale:

FIGO Staging for Endometrial Cancer:
Stage I: Confined to the uterus
Stage II: Involves the cervix
Stage III: Spread to lymph nodes or adnexa
Stage IV: 4A = Tumor invades the bladder or bowel mucosa; 4B = Distant metastases (lungs, liver, etc.)

109
Q
  1. A 29-year-old woman with hypertension and diabetes is diagnosed with Stage I endometrial carcinoma. What is the best therapeutic management?
    A. Chemotherapy
    B. Radiation therapy
    C. Progesterone therapy
    D. Surgery
A

D. Surgery

πŸ”Ž Rationale:

Stage I endometrial cancer is best treated with Total Abdominal Hysterectomy with Bilateral Salpingo-Oophorectomy (TAHBSO).
Adjuvant therapy (radiation or chemotherapy) is considered in high-risk Stage I cases.
Progesterone therapy is an option for fertility preservation but is not the standard treatment.

110
Q
  1. A 48-year-old woman with Endometrial Stromal Sarcoma (ESS) has Estrogen-Progesterone receptors. Which of the following best describes the role of these receptors in cancer development?
    A. They function as oncogenes that suppress tumor growth.
    B. They allow cancer cells to attach to surrounding tissues.
    C. They are proteins that bind to hormones and promote tumor growth.
    D. They directly cause mutations in DNA.
A

C. They are proteins that bind to hormones and promote tumor growth.

Rationale:

Receptors are proteins found in or on cells that bind to hormones in the blood.
Normal breast and some cancer cells (including ESS) have estrogen and progesterone receptors, which promote tumor growth when these hormones bind.
ESS is often hormone-sensitive, making hormonal therapy a treatment option.

111
Q
  1. A 41-year-old woman who underwent total hysterectomy with bilateral salpingo-oophorectomy (THBSO) was diagnosed with a tumor with >5 mitoses per high-power field. What is the most likely diagnosis?
    A. Leiomyoma
    B. Leiomyosarcoma
    C. Endometrial Stromal Sarcoma
    D. Adenosarcoma
A

B. Leiomyosarcoma

Rationale:

Leiomyosarcoma (LMS) is a rare and aggressive smooth muscle tumor that arises in hollow organs such as the uterus.
A key diagnostic feature is the presence of >5 mitoses per high-power field (HPF) in histology.
Unlike leiomyomas (benign fibroids), LMS is malignant and rapidly grows.

112
Q
  1. What is an individual’s lifetime risk of developing ovarian cancer?
    A. 0.5%
    B. 1.3%
    C. 5%
    D. 10%
A

B. 1.3%

Rationale:

The general lifetime risk of ovarian cancer in women is 1.3%.
However, BRCA1/BRCA2 mutations or Lynch syndrome (HNPCC) increase the risk significantly.

113
Q
  1. An endometrial mass composed of both carcinomatous and sarcomatous components is classified as which type of tumor?
    A. Endometrial Stromal Sarcoma
    B. Leiomyosarcoma
    C. Carcinosarcoma
    D. Adenomyoma
A

C. Carcinosarcoma

Rationale:

Carcinosarcoma (also known as malignant mixed MΓΌllerian tumor) contains both carcinomatous (epithelial) and sarcomatous (mesenchymal) components.
These tumors behave aggressively and are classified under high-risk endometrial cancers.

114
Q
  1. The majority of ovarian cancers arise from which of the following?
    A. Genetic mutations (BRCA1/BRCA2)
    B. Hereditary syndromes
    C. Sporadic events
    D. Hormonal exposure
A

C. Sporadic events

Rationale:

Most ovarian cancers occur sporadically, meaning they arise from random mutations rather than inherited genetic predispositions.
Only 10-15% of ovarian cancers are hereditary (e.g., BRCA mutations, Lynch Syndrome).

115
Q
  1. Which theory states that ovarian cancer is caused by repeated monthly damage due to ovulation?
    A. Incessant Ovulation Hypothesis
    B. Estrogen-Progesterone Hypothesis
    C. Ovarian Stem Cell Theory
    D. Serous Tubal Origin Theory
A

A. Incessant Ovulation Hypothesis

Rationale:

The incessant ovulation hypothesis suggests that continuous ovulation causes repeated epithelial damage to the ovary, leading to DNA mutations and increased cancer risk.
Protective factors include pregnancy, OCP use, and breastfeeding (as they reduce ovulation).

116
Q
  1. The Fallopian Tube Theory of Ovarian Cancer states that a precursor lesion in the fimbriae contributes to high-grade serous carcinoma. What is this lesion called?
    A. Endosalpingiosis
    B. Mesenchymal atypical cells
    C. Serous Tubal Intraepithelial Carcinoma (STIC)
    D. Papillary Serous Carcinoma
A

C. Serous Tubal Intraepithelial Carcinoma (STIC)

Rationale:

Recent research suggests that most high-grade serous ovarian cancers originate from the fallopian tubes, not the ovary itself.
The precursor lesion is called β€œSerous Tubal Intraepithelial Carcinoma (STIC),” found in the fimbrial end of the fallopian tube.
STIC lesions have TP53 mutations, which are early genetic changes leading to ovarian carcinoma.

117
Q
  1. All of the following cancers originate from the ovarian stroma, EXCEPT:
    A. Serous Adenocarcinoma
    B. Epidermoid Carcinoma
    C. Sertoli-Leydig Cell Tumor
    D. Mucinous Carcinoma
A

A. Serous Adenocarcinoma

Rationale:

Sex cord-stromal tumors originate from ovarian stroma, including Granulosa Cell Tumors and Sertoli-Leydig Cell Tumors.
Serous adenocarcinoma and mucinous carcinoma originate from the ovarian epithelium (not the stroma).
Epidermoid carcinoma is a rare ovarian malignancy that can arise from teratomas.

118
Q
  1. Which of the following is associated with a decreased risk of ovarian cancer?
    A. Smoking
    B. Late menopause
    C. Long-term oral contraceptive (OCP) use
    D. Endometriosis
A

C. Long-term oral contraceptive (OCP) use

Rationale:

OCP use is one of the most protective factors against ovarian cancer (50% risk reduction for up to 25 years).
Late menopause increases risk (more ovulation = more DNA damage).
Endometriosis is associated with an increased risk of ovarian cancer, particularly clear cell and endometrioid types.
Smoking does not protect against ovarian cancer and is linked to mucinous ovarian carcinoma.

119
Q
  1. BRCA1 and BRCA2 mutations are associated with which hereditary cancer syndrome?
    A. None
    B. Hereditary Breast and Ovarian Cancer Syndrome
    C. Li-Fraumeni Syndrome
    D. Lynch Syndrome
A

B. Hereditary Breast and Ovarian Cancer Syndrome

Rationale:

BRCA1 and BRCA2 mutations significantly increase the risk of both breast and ovarian cancer.
Li-Fraumeni Syndrome is associated with TP53 mutations.
Lynch Syndrome is linked to DNA mismatch repair gene mutations (MLH1, MSH2, MSH6, PMS2, EPCAM) and is primarily associated with colorectal and endometrial cancer.

120
Q
  1. A 71-year-old woman presents with abdominal enlargement and ascites. What is the next best step in diagnosis?
    A. X-ray
    B. Transvaginal Ultrasound
    C. Pelvic Examination
    D. MRI
A

B. Transvaginal Ultrasound

Rationale:

Transvaginal ultrasound (TVUS) is the most useful imaging modality for ovarian masses and has the highest sensitivity.
Pelvic examination can detect a large ovarian mass but is not accurate for small ones.
X-ray is not the primary diagnostic tool for ovarian cancer.
MRI may be used for further characterization but is not the first-line test.

121
Q
  1. A patient presents with an ovarian cystic mass that has no solid components and no blood flow. What is the most likely diagnosis?
    A. Malignant ovarian tumor
    B. Benign ovarian mass
    C. Serous cystadenocarcinoma
    D. Dysgerminoma
A

B. Benign ovarian mass

Rationale:

A purely cystic ovarian mass without solid components and without blood flow is most likely benign.
Angiogenesis (new blood vessel formation) is a characteristic of malignant tumors, which is why malignant masses typically have increased blood flow on Doppler imaging.
Serous cystadenocarcinoma and dysgerminoma often show solid components and vascularization.

122
Q
  1. What is the most common ovarian malignancy among women of reproductive age?
    A. Mucinous Carcinoma
    B. Serous Adenocarcinoma
    C. Clear Cell Carcinoma
    D. Granulosa Cell Tumor
A

B. Serous Adenocarcinoma

Rationale:

Serous adenocarcinoma is the most common ovarian malignancy in reproductive-age women.
It accounts for the majority of epithelial ovarian cancers (EOCs).
Mucinous carcinoma is less common and more frequently seen in older women.
Clear cell carcinoma is associated with endometriosis.

123
Q
  1. What is the most common malignant germ cell tumor of the ovary?
    A. Yolk Sac Tumor
    B. Immature Teratoma
    C. Dysgerminoma
    D. Choriocarcinoma
A

C. Dysgerminoma

Rationale:

Dysgerminoma is the most common malignant germ cell tumor and accounts for 45% of malignant ovarian germ cell tumors.
Yolk sac tumors are the second most common and produce AFP.
Immature teratomas are rare but occur in young women.
Choriocarcinoma of ovarian origin is extremely rare.

124
Q
  1. During surgery, the tissue surrounding the tumor breaks, potentially spreading cancer to the abdomen and pelvis. What is the correct FIGO stage?
    A. Stage IA
    B. Stage IC1
    C. Stage IC2
    D. Stage II
A

B. Stage IC1

Rationale:

Stage IC1 refers to surgical spill, meaning that the tumor was confined to the ovary but ruptured during surgery, potentially allowing cancer cells to spread into the abdomen and pelvis.
Stage IC2 refers to capsule rupture before surgery or tumor on the ovarian surface.
Stage II and beyond indicate extraovarian spread.

125
Q
  1. A patient with large abdominal masses and pleural effusion in one lung is classified under which FIGO stage?
    A. Stage IIIA
    B. Stage IIIB
    C. Stage IVA
    D. Stage IVB
A

C. Stage IVA

Rationale:

Stage IVA is defined as pleural effusion with positive cytology, meaning cancer cells are found in the fluid.
Stage IVB indicates distant metastases beyond the peritoneal cavity.

126
Q
  1. What is the main histologic criterion that differentiates low malignant potential (LMP) ovarian tumors from invasive carcinoma?
    A. Nuclear atypia
    B. Absence of stromal invasion
    C. High mitotic rate
    D. Necrosis and hemorrhage
A

B. Absence of stromal invasion

Rationale:

Low malignant potential (LMP) tumors lack stromal invasion, distinguishing them from invasive carcinomas.
LMP tumors can still show nuclear atypia but do not invade surrounding tissue.

127
Q
  1. A 50-year-old woman underwent total hysterectomy with bilateral salpingo-oophorectomy (THBSO) for a low malignant potential (LMP) tumor. What is the best management?
    A. Chemotherapy
    B. Radiation therapy
    C. Regular follow-up
    D. Hormonal therapy
A

C. Regular follow-up

Rationale:

LMP tumors generally do not require chemotherapy unless they progress to invasive carcinoma.
Regular follow-up is sufficient to monitor for recurrence.

128
Q
  1. High-grade serous carcinoma arises from the fimbriated end of the fallopian tube through which molecular pathway?
    A. BRCA1/BRCA2
    B. TP53 mutation pathway
    C. PI3K-AKT pathway
    D. Lynch syndrome pathway
A

B. TP53 mutation pathway

Rationale:

High-grade serous ovarian carcinoma (HGSC) originates from serous tubal intraepithelial carcinoma (STIC) in the fimbriae of the fallopian tube.
The TP53 mutation is a hallmark of HGSC and is an early genetic event.

129
Q
  1. What is the third most common genital tract malignancy in the Philippines?
    A. Cervical cancer
    B. Ovarian cancer
    C. Endometrial cancer
    D. Vaginal cancer
A

C. Endometrial cancer

Rationale:

Endometrial carcinoma is the third most common gynecologic malignancy in the Philippines.
Cervical cancer remains the most common, followed by ovarian cancer.

130
Q
  1. A 53-year-old woman undergoing surgery for ovarian cancer was diagnosed with endometriosis 10 years ago. What malignancy is associated with this history?
    A. Serous carcinoma
    B. Clear cell carcinoma
    C. Endometrioid adenocarcinoma
    D. Mucinous carcinoma
A

B. Clear cell carcinoma

131
Q
  1. A 46-year-old woman presents with abdominal enlargement, a benign ovarian tumor, and pleural effusion. What is the likely diagnosis?
    A. Serous cystadenoma
    B. Thecoma
    C. Fibroma
    D. Meigs Syndrome
A

D. Meigs Syndrome

Rationale:

Meigs Syndrome is characterized by a benign ovarian tumor (often a fibroma), ascites, and pleural effusion.
Symptoms resolve after tumor removal.

132
Q
  1. What is the most common germ cell tumor, characterized by the ability to produce adult tissues?
    A. Dysgerminoma
    B. Immature teratoma
    C. Dermoid cyst (Mature cystic teratoma)
    D. Yolk sac tumor
A

C. Dermoid cyst (Mature cystic teratoma)

Rationale:

Dermoid cysts contain mature tissues from all three germ layers (ectoderm, mesoderm, and endoderm).
They can include hair, teeth, and sebaceous material.

133
Q
  1. A 28-year-old woman presents with urinary frequency and a 9 cm benign ovarian tumor. What is the best treatment?
    A. Oophorectomy
    B. Oophorocystectomy
    C. Chemotherapy
    D. Radiation therapy
A

B. Oophorocystectomy

Rationale:

Oophorocystectomy (cyst removal) is the preferred treatment for benign ovarian tumors, preserving ovarian function.
Oophorectomy (removal of the entire ovary) is reserved for malignant cases.

134
Q
  1. Which histologic ovarian cancer type is characterized by β€œhobnail cells”?
    A. Clear cell carcinoma
    B. Serous carcinoma
    C. Mucinous carcinoma
    D. Endometrioid carcinoma
A

A. Clear cell carcinoma

Rationale:

Clear cell carcinoma is histologically identified by hobnail cells, which are tumor cells with nuclei protruding into the glandular lumen.
It is associated with endometriosis.

135
Q
  1. What is the standard treatment for ovarian cancer?
    A. Surgery
    B. Chemotherapy
    C. Radiation therapy
    D. Targeted therapy
A

B. Chemotherapy

Rationale:

Platinum-based chemotherapy (Carboplatin + Paclitaxel) is the standard treatment for advanced ovarian cancer.
Surgery is also crucial, but chemotherapy is necessary for most cases beyond Stage I.

136
Q
  1. What is the most frequently found tissue in an immature teratoma?
    A. Bone
    B. Neural tissue
    C. Cartilage
    D. Skin
A

B. Neural tissue

Rationale:

Immature teratomas contain immature neural elements, which determine tumor grade.
Higher amounts of immature neural tissue indicate a more aggressive tumor.

137
Q
  1. A 15-year-old girl presents with abdominal enlargement. What tumor markers should be ordered?
    A. CA-125
    B. hCG, AFP, LDH, CA-125
    C. BRCA1/2
    D. CEA
A

B. hCG, AFP, LDH, CA-125

Rationale:

In young patients with abdominal masses, germ cell tumors are suspected.
Tumor markers (hCG, AFP, LDH) help in diagnosis.

138
Q
  1. The prognosis for patients with immature teratoma depends on which factor?
    A. Amount of mature and immature tissue
    B. Age at diagnosis
    C. Stage and grade
    D. Tumor location
A

C. Stage and grade

Rationale:

The prognosis of immature teratomas depends on both FIGO stage and tumor grade.
Higher tumor grade (presence of more immature neural tissue) is associated with a worse prognosis.
Patients with Stage I disease generally have an excellent prognosis.

139
Q
  1. This rare benign ovarian tumor contains thyroid tissue in more than 50% of the overall tumor.
    A. Mature cystic teratoma
    B. Struma ovarii
    C. Dysgerminoma
    D. Brenner tumor
A

B. Struma ovarii

Rationale:

Struma ovarii is a rare monodermal teratoma that contains functional thyroid tissue (>50%).
It can cause hyperthyroidism due to excess thyroid hormone production.
It is usually benign but can rarely become malignant.

140
Q
  1. A 28-year-old nulligravid woman presents with abdominal pain. A 10x9 cm solid ovarian mass is found. What is the most common malignant neoplasm detected in pregnancy?
    A. Endometrioid carcinoma
    B. Yolk sac tumor
    C. Dysgerminoma
    D. Serous adenocarcinoma
A

C. Dysgerminoma

Rationale:

Dysgerminoma is the most common malignant ovarian tumor detected during pregnancy.
It is a germ cell tumor and is highly curable with fertility-sparing surgery and chemotherapy.
It is associated with increased LDH as a tumor marker.

141
Q
  1. What histologic feature is characteristic of yolk sac tumors?
    A. Call-Exner bodies
    B. Psammoma bodies
    C. Schiller-Duval bodies
    D. Hobnail cells
A

C. Schiller-Duval bodies

Rationale:

Schiller-Duval bodies are the pathognomonic histologic feature of yolk sac tumors.
They resemble glomeruli and are seen under microscopic examination.
Yolk sac tumors also produce AFP as a tumor marker.

142
Q
  1. What is the tumor marker of choice for granulosa cell tumors?
    A. CA-125
    B. Ξ²-hCG
    C. Inhibin
    D. LDH
A

C. Inhibin

Rationale:

Granulosa cell tumors secrete inhibin, which is the most reliable tumor marker for diagnosis and monitoring.
They can also cause hyperestrogenism, leading to endometrial hyperplasia or carcinoma.

143
Q
  1. A 22-year-old nulligravid woman has a tumor involving the left ovary with visible tumor nodules. What is the next step?
    A. Hysterectomy with bilateral salpingo-oophorectomy
    B. Left salpingo-oophorectomy (LSO)
    C. Chemotherapy
    D. Cancer staging
A

D. Cancer staging

Rationale:

Cancer staging is crucial before definitive treatment in young women with ovarian tumors.
Surgical staging includes omentectomy, peritoneal washings, lymph node sampling, and tumor resection.

144
Q
  1. A 51-year-old G2 woman presents with abdominal enlargement and vaginal bleeding. What is the most likely ovarian tumor?
    A. Serous cystadenoma
    B. Mucinous carcinoma
    C. Granulosa cell tumor
    D. Thecoma
A

C. Granulosa cell tumor

Rationale:

Granulosa cell tumors produce estrogen, which can cause abnormal uterine bleeding in postmenopausal women.
They are often associated with endometrial hyperplasia or carcinoma.
Inhibin is the tumor marker of choice for monitoring.

145
Q
  1. Which statement is true about hydrops in fallopian tube cancer?
    A. It is associated with high fever and infection.
    B. It is characterized by cyclical pain relieved by the passage of vaginal discharge.
    C. It is always associated with Latzko’s triad.
    D. It does not cause any symptoms.
A

B. It is characterized by cyclical pain relieved by the passage of vaginal discharge.

Rationale:

Hydrops tubae profluens refers to pain relieved by the sudden passage of fluid per vagina.
It occurs due to intermittent distension of the fallopian tube from accumulated fluid.

146
Q
  1. Which of the following is true about Latzko’s triad?
    A. It is pathognomonic for fallopian tube cancer but is present in only 15% of cases.
    B. It includes postmenopausal bleeding, fever, and pelvic mass.
    C. It is seen in over 90% of cases of fallopian tube carcinoma.
    D. It is used to diagnose ovarian cancer.
A

A. It is pathognomonic for fallopian tube cancer but is present in only 15% of cases.

Rationale:

Latzko’s triad consists of watery vaginal discharge, colicky lower abdominal pain, and a pelvic mass.
It is characteristic of fallopian tube cancer but is seen in less than 15% of cases.

147
Q
  1. The following criteria should be satisfied for diagnosing an ovarian tumor, EXCEPT:
    A. The uterus and ovaries are normal in size.
    B. The tumor has a solid component.
    C. The fallopian tube is involved.
    D. The mass is detected on imaging.
A

A. The uterus and ovaries are normal in size.

Rationale:

Ovarian tumors typically present with an enlarged ovary, which is why normal-sized ovaries and uterus would not be consistent with the diagnosis of an ovarian tumor.
Other key criteria include tumor characteristics such as solid components, imaging findings, and sometimes involvement of adjacent structures.

148
Q
  1. The following is true about ovarian rupture during surgery, EXCEPT:
    A. Tumor rupture worsens prognosis by spreading cancer cells.
    B. Spontaneous tumor rupture has a poorer prognosis than intraoperative rupture.
    C. Ruptured tumors are staged as FIGO IC1.
    D. Tumor rupture improves survival compared to intact tumors.
A

D. Tumor rupture improves survival compared to intact tumors.

Rationale:

Tumor rupture allows cancer cells to spread within the peritoneal cavity, worsening prognosis.
Spontaneous rupture before surgery has a poorer prognosis than surgical rupture (IC1 staging).
Intact tumors have a better survival rate than ruptured tumors.

149
Q
  1. A 51-year-old G2 woman presents with abdominal enlargement and vaginal bleeding. What is the most likely ovarian tumor?
    A. Serous cystadenocarcinoma
    B. Mucinous carcinoma
    C. Granulosa cell tumor
    D. Thecoma
A

C. Granulosa cell tumor

Rationale:

Granulosa cell tumors are estrogen-secreting tumors that can cause abnormal uterine bleeding, particularly in postmenopausal women.
They are also associated with endometrial hyperplasia and endometrial carcinoma.

150
Q
  1. A young patient undergoing surgery for epithelial ovarian cancer (EOC) may retain her uterus and contralateral ovary if:
    A. The tumor is confined to one ovary.
    B. There is evidence of capsule rupture.
    C. The tumor is poorly differentiated.
    D. The cancer has spread to the peritoneum.
A

A. The tumor is confined to one ovary.

Rationale:

In young patients desiring fertility preservation, conservative surgery is possible if the tumor is:
Confined to one ovary (Stage IA)
No capsular rupture
Well-differentiated
More advanced disease requires complete surgical removal.

151
Q
  1. What is the most common histologic type found in fallopian tube cancer?
    A. Serous adenocarcinoma
    B. Endometrioid carcinoma
    C. Mucinous carcinoma
    D. Clear cell carcinoma
A

A. Serous adenocarcinoma

Rationale:

High-grade serous carcinoma is the most common histologic type of fallopian tube cancer.
It originates from the fimbrial end of the fallopian tube and follows the TP53 mutation pathway.

152
Q
  1. A 48-year-old healthy postmenopausal woman undergoes a Pap smear, which reveals atypical glandular cells. She has no history of abnormal Pap smears. What is the next best step?
    A. Colposcopy, endometrial sampling, and dilatation & curettage
    B. Repeat Pap smear in 6 months
    C. Transvaginal ultrasound
    D. HPV co-testing
    E. Observation
A

A. Colposcopy, endometrial sampling, and dilatation & curettage

Rationale:

Atypical glandular cells (AGC) in a postmenopausal woman may indicate endometrial or cervical pathology.
Colposcopy, endometrial biopsy, and D&C are recommended to rule out malignancy.

153
Q
  1. A 15-year-old girl presents with breast enlargement and vaginal bleeding. Imaging reveals an 8 cm pelvic mass. What is the most likely etiology?
    A. Granulosa-theca cell tumor
    B. Dysgerminoma
    C. Yolk sac tumor
    D. Serous cystadenoma
    E. Mature teratoma
A

A. Granulosa-theca cell tumor

Rationale:

Granulosa-theca cell tumors secrete estrogen, leading to precocious puberty in young patients.
Other tumors do not typically produce hormones.

154
Q
  1. A 25-year-old woman is noted to have a 4 cm simple ovarian cyst on ultrasound. She has no pain, nausea, or vomiting. What is the next step?
    A. Expectant management
    B. Immediate surgical removal
    C. Chemotherapy
    D. Hormonal therapy
    E. Laparotomy
A

A. Expectant management

Rationale:

Simple ovarian cysts <5 cm in premenopausal women are usually functional and resolve spontaneously.
Follow-up ultrasound in 6–8 weeks is recommended.

155
Q
  1. What is the best treatment for a suspected desmoid tumor in an 18-year-old nulliparous woman?
    A. Ovarian cystectomy
    B. Hysterectomy
    C. Chemotherapy
    D. Radiation therapy
    E. Tamoxifen
A

A. Ovarian cystectomy

Rationale:

Desmoid tumors are benign and can be managed with ovarian cystectomy.

156
Q
  1. A 55-year-old woman presents with an abdominal mass and increased abdominal girth. On examination, there is shifting dullness and a fluid wave. What is the most likely malignancy?
    A. Ovarian cancer
    B. Endometrial cancer
    C. Cervical cancer
    D. Uterine sarcoma
    E. Breast cancer
A

A. Ovarian cancer

Rationale:

Ascites and an adnexal mass are highly suggestive of ovarian cancer.
Ovarian malignancies frequently present with peritoneal spread.

157
Q
  1. Which of the following are epithelial ovarian tumors?
    A. Endometrioid tumor
    B. Serous cystadenocarcinoma
    C. Mucinous cystadenoma
    D. Clear cell carcinoma
    E. All of the above
A

: E. All of the above

Rationale:

Epithelial ovarian tumors include serous, mucinous, endometrioid, and clear cell carcinomas.

158
Q
  1. Which of the following are germ cell tumors of the ovary?
    A. Dysgerminoma
    B. Yolk sac tumor
    C. Immature teratoma
    D. Embryonal carcinoma
    E. All of the above
A

E. All of the above

Rationale:

Germ cell tumors originate from primitive germ cells and include dysgerminoma, yolk sac tumor, teratomas, and embryonal carcinoma.

159
Q
  1. Which of the following are sex cord-stromal tumors of the ovary?
    A. Sertoli-Leydig tumor
    B. Granulosa cell tumor
    C. Thecoma
    D. Fibroma
    E. All of the above
A

E. All of the above

Rationale:

Sex cord-stromal tumors originate from gonadal stroma and include granulosa cell tumors, Sertoli-Leydig tumors, and fibromas.

160
Q
  1. Which ovarian tumor is treated with laparoscopic ovarian cystectomy in a premenopausal woman?
    A. Endometriotic cyst
    B. Serous cystadenoma
    C. Mucinous cystadenoma
    D. Dermoid cyst
    E. All of the above
A

E. All of the above

Rationale:

Benign ovarian cysts in young women can be removed via laparoscopic cystectomy.

161
Q
  1. A 40-year-old G3P3 presents with 3 months of abdominal enlargement. Imaging suggests ovarian cancer with possible omental metastasis. What type of surgery is recommended?
    A. Total abdominal hysterectomy with bilateral salpingo-oophorectomy (TAH-BSO) and cancer staging
    B. Ovarian cystectomy
    C. Unilateral salpingo-oophorectomy
    D. Omentectomy alone
    E. Chemotherapy first
A

A. TAH-BSO and cancer staging

Rationale:

Ovarian cancer treatment requires complete cytoreduction and staging surgery.

162
Q
  1. How is carcinoma of the ovary staged?
    A. Surgically
    B. Clinically
    C. By tumor markers
    D. By imaging alone
    E. By biopsy
A

A. Surgically

Rationale:

Ovarian cancer is staged surgically according to FIGO criteria.

163
Q
  1. CA-125 is elevated in which type of ovarian cancer?
    A. Serous cystadenocarcinoma
    B. Mucinous carcinoma
    C. Clear cell carcinoma
    D. Endometrioid carcinoma
    E. All of the above
A

A. Serous cystadenocarcinoma

Rationale:

CA-125 is primarily used as a tumor marker for serous ovarian carcinoma.

164
Q
  1. Which of the following is true regarding CA-125?
    A. Serial values are used to assess response to treatment in malignant ovarian tumors
    B. It is useful for screening in asymptomatic women
    C. It is highly specific for ovarian cancer
    D. It is not elevated in any benign conditions
    E. It is only used in postmenopausal women
A

A. Serial values are used to assess response to treatment in malignant ovarian tumors

Rationale:

CA-125 is not used for screening but is useful for monitoring response to treatment in ovarian cancer.

165
Q
  1. Which of the following is true regarding tumor markers in ovarian tumors?
    A. Inhibin levels are elevated in granulosa cell tumors
    B. Ξ²-hCG is a marker for mucinous carcinoma
    C. CA-125 is specific for all ovarian cancers
    D. LDH is elevated in epithelial ovarian cancer
    E. AFP is specific for dysgerminomas
A

A. Inhibin levels are elevated in granulosa cell tumors

Rationale:

Granulosa cell tumors secrete inhibin, which serves as a tumor marker.
Ξ²-hCG is associated with choriocarcinoma, not mucinous carcinoma.

166
Q
  1. What type of Gestational Trophoblastic Disease (GTD) is characterized by an extensively edematous placenta?
    A. Complete Hydatidiform Mole
    B. Partial Hydatidiform Mole
    C. Invasive Mole
    D. Placental Site Trophoblastic Tumor
A

A. Complete Hydatidiform Mole

Rationale:
Complete hydatidiform mole (CHM) is characterized by extensive hydropic degeneration of chorionic villi, giving the placenta a grape-like appearance due to edematous vesicles. There is no fetal tissue present, and trophoblastic proliferation is diffuse.

167
Q
  1. What type of GTD is deemed malignant due to its marked penetration and destruction of the myometrium, with the ability to metastasize?
    A. Complete Hydatidiform Mole
    B. Partial Hydatidiform Mole
    C. Invasive Mole
    D. Placental Site Trophoblastic Tumor
A

C. Invasive Mole

Rationale:
Invasive mole is a type of Gestational Trophoblastic Neoplasia (GTN) that invades the myometrium, causing destruction of uterine tissues and has potential for metastasis. It most commonly arises after a complete mole.

168
Q
  1. Which type of hydatidiform mole has the highest risk of progressing to Gestational Trophoblastic Neoplasia (GTN)?
    A. Complete Hydatidiform Mole
    B. Partial Hydatidiform Mole
    C. Both A and B
    D. None of the above
A

A. Complete Hydatidiform Mole

Rationale:
GTN most commonly arises after a complete mole (15-20% risk) compared to a partial mole (1-5% risk). Complete moles lack fetal tissue and exhibit diffuse trophoblastic proliferation, increasing the risk of malignancy.

169
Q
  1. Which type of mole results from androgenesis?
    A. Complete Hydatidiform Mole
    B. Partial Hydatidiform Mole
    C. Both A and B
    D. None of the above
A

A. Complete Hydatidiform Mole

Rationale:
Complete hydatidiform mole results from androgenesis, where the fertilized ovum has only paternal chromosomes. This occurs when an empty egg is fertilized by a sperm that duplicates its genome (46,XX) or when two sperm fertilize an empty ovum (46,XY or 46,XX from dispermy).

170
Q
  1. Which type of hydatidiform mole does not involve the entire placenta and contains fetal embryonic tissue?
    A. Complete Hydatidiform Mole
    B. Partial Hydatidiform Mole
    C. Both A and B
    D. None of the above
A

B. Partial Hydatidiform Mole

Rationale:
Partial hydatidiform moles result from triploidy (69,XXY, XXX, or XYY) and contain fetal embryonic tissue, unlike complete moles, which lack fetal development. Only some villi are edematous, distinguishing it from the diffuse involvement of a complete mole.

171
Q
  1. Which type of mole is characterized by a cluster of vesicles with variable dimensions?
    A. Complete Hydatidiform Mole
    B. Partial Hydatidiform Mole
    C. Both A and B
    D. None of the above
A

A. Complete Hydatidiform Mole

Rationale:
The hallmark of a complete hydatidiform mole is the presence of multiple hydropic vesicles (β€œgrape-like clusters”), which are enlarged, edematous chorionic villi.

172
Q
  1. A 18-year-old G1P0 presents with vaginal bleeding after 4 months of amenorrhea. Physical exam reveals an enlarged uterus (20-22 weeks AOG) with no fetal heart tones. What is the most likely diagnosis?
    A. Ectopic Pregnancy
    B. Threatened Abortion
    C. Molar Pregnancy
    D. A & B
A

C. Molar Pregnancy

Rationale:
Key features of molar pregnancy:

Uterus size larger than gestational age
No fetal heart tones
Vaginal bleeding
Ξ²-hCG markedly elevated
Ultrasound: β€œSnowstorm appearance”

173
Q
  1. A negative pregnancy test despite very high Ξ²-hCG levels is due to what phenomenon?
    A. Braxton Hicks Effect
    B. Ferguson Effect
    C. Ferning Effect
    D. Hook Effect
A

D. Hook Effect

Rationale:
The Hook Effect occurs when extremely high levels of Ξ²-hCG oversaturate the assay antibodies, leading to falsely negative or low pregnancy test results. It is commonly seen in molar pregnancies and GTN.

174
Q
  1. What is the sonographic appearance of a molar pregnancy?
    A. Snowstorm pattern
    B. Ground-glass appearance
    C. Double-ring sign
    D. Railroad track sign
A

A. Snowstorm pattern

Rationale:
A complete hydatidiform mole appears as a heterogeneous, echogenic mass with numerous anechoic cystic spaces resembling a snowstorm on ultrasound.

175
Q
  1. Immunostaining used to differentiate complete moles from partial moles?
    A. p53
    B. p16
    C. p57 KIP2
    D. Ki-67
A

C. p57 KIP2

Rationale:

p57 KIP2 is absent in complete moles because it is maternally expressed.
Partial moles and normal pregnancy express p57 KIP2, helping in diagnosis.

176
Q
  1. A 22-year-old G1P0 presents at 14 weeks with vaginal bleeding and is diagnosed with a molar pregnancy. What is the preferred treatment?
    A. Methotrexate
    B. Hysterectomy
    C. Suction Curettage
    D. Wait and monitor
A

C. Suction Curettage

Rationale:
Suction curettage is the gold standard for managing hydatidiform moles, especially in women who want future pregnancies. It is less invasive than a hysterectomy.

177
Q
  1. A 40-year-old G6P5 (5-0-0-5) at 14 weeks is diagnosed with a molar pregnancy. What is the most appropriate management?
    A. Methotrexate
    B. Hysterectomy
    C. Suction Curettage
    D. Observation
A

B. Hysterectomy

Rationale:
In multiparous women or those β‰₯40 years old, hysterectomy is often preferred because:

Higher risk of GTN (due to age and prior pregnancies).
Definitive treatment (no risk of recurrence in the uterus).
Avoids need for close Ξ²-hCG monitoring.

178
Q
  1. A 37-year-old G2P1 (1001) at 18 weeks is diagnosed with a molar pregnancy. What is the preferred treatment?
    A. Methotrexate
    B. Hysterectomy
    C. Suction Curettage
    D. Expectant Management
A

C. Suction Curettage

Rationale:
Suction curettage is the gold standard for molar pregnancy treatment in women who want future pregnancies.

179
Q
  1. A 35-year-old G5P4 (4014) is in her 4th week post-suction curettage for a molar pregnancy. How should this be considered in the TPAL system?
    A. G5P4 (4014)
    B. G5P5 (5014)
    C. G6P5 (5015)
    D. G5P4 (4015)
A

B. G5P5 (5014)

Rationale:

A molar pregnancy is classified as an abortion in TPAL.
The patient’s parity (P) increases by 1 after evacuation.

180
Q
  1. A patient 4 weeks postpartum after curettage for molar pregnancy presents to the ER for irregular bleeding. PE reveals a uterus undergoing normal involution. What is the most likely diagnosis?
    A. Retained Products of Conception
    B. Incomplete Abortion
    C. Gestational Trophoblastic Neoplasia (GTN)
    D. Endometritis
A

C. Gestational Trophoblastic Neoplasia (GTN)

Rationale:

GTN should be suspected in a post-molar pregnancy patient with persistent or irregular bleeding.
Diagnosis is confirmed by persistently elevated or plateauing Ξ²-hCG levels.

181
Q
  1. When is the best time to obtain Ξ²-hCG levels post-evacuation of a molar pregnancy?
    A. 24 hours
    B. 48 hours
    C. 72 hours
    D. 1 week
A

B. 48 hours

Rationale:
Ξ²-hCG levels double approximately every 48 hours in a progressing pregnancy, making this the best time to assess decline post-evacuation.

182
Q
  1. What is the median time for Ξ²-hCG resolution in a complete molar pregnancy?
    A. 6 weeks
    B. 7 weeks
    C. 9 weeks
    D. 12 weeks
A

C. 9 weeks

Rationale:

Complete moles take longer to resolve than partial moles.
The median time for complete mole Ξ²-hCG normalization is 9 weeks.

183
Q
  1. Once Ξ²-hCG becomes undetectable, how long should monthly Ξ²-hCG measurements continue?
    A. 3 months
    B. 6 months
    C. 9 months
    D. 12 months
A

B. 6 months

Rationale:

After Ξ²-hCG normalization, monthly monitoring for 6 months is required to detect GTN early.

184
Q
  1. What form of contraception should NOT be used immediately after molar pregnancy evacuation?
    A. Oral Contraceptive Pills (OCP)
    B. Injectable Medroxyprogesterone Acetate
    C. Progestin Implant
    D. Intrauterine Device (IUD)
A

D. Intrauterine Device (IUD)

Rationale:

IUDs are avoided until Ξ²-hCG is undetectable due to the risk of uterine perforation in an invasive mole.
Hormonal contraceptives are preferred for reliable pregnancy prevention.

185
Q
  1. Which of the following is NOT a risk factor for Gestational Trophoblastic Neoplasia (GTN)?
    A. Ξ²-hCG level > 100,000 mIU/mL
    B. Uterine size larger than gestational age
    C. Theca-lutein cysts < 6 cm
    D. Slow decline in Ξ²-hCG levels
A

C. Theca-lutein cysts < 6 cm

Rationale:

Large theca-lutein cysts (>6 cm) are a risk factor for GTN, but smaller cysts (<6 cm) are not.

186
Q
  1. All of the following are criteria for the diagnosis of GTN EXCEPT?
    A. Histologic criteria for molar pregnancy
    B. Plateau of serum Ξ²-hCG for 3 weeks
    C. Serum Ξ²-hCG remains detectable for 6 months
    D. Rise of serum Ξ²-hCG by >10% over 3 readings
A

A. Histologic criteria for molar pregnancy

Rationale:

GTN is diagnosed based on persistently elevated or rising Ξ²-hCG levels, not histology.

187
Q
  1. In the WHO Prognostic Scoring System for GTN, what score is considered high risk?
    A. 5
    B. 6
    C. 7
    D. 10
A

C. 7

Rationale:
A WHO score β‰₯7 is considered high risk, requiring multi-agent chemotherapy.

188
Q
  1. If GTN involves the upper vagina, what is the WHO Prognostic Score?
    A. 1
    B. 2
    C. 3
    D. 4
A

B. 2

Rationale:
Metastases to the upper vagina = Score 2 in WHO GTN Prognostic Scoring.

189
Q
  1. If GTN involves the lungs, what is the WHO Prognostic Score?
    A. 1
    B. 2
    C. 3
    D. 4
A

C. 3

Rationale:
Metastases to the lungs = Score 3 in WHO GTN Prognostic Scoring.

190
Q
  1. Which GTN arises from intermediate trophoblasts at the placental site?
    A. Choriocarcinoma
    B. Invasive Mole
    C. Placental Site Trophoblastic Tumor (PSTT)
    D. Epitheloid Trophoblastic Tumor (ETT)
A

C. Placental Site Trophoblastic Tumor (PSTT)

Rationale:

PSTT originates from intermediate trophoblasts at the placental site, has low Ξ²-hCG levels, and is chemotherapy-resistant.

190
Q
  1. What is the most common Gestational Trophoblastic Neoplasia (GTN) that frequently follows a hydatidiform mole?
    A. Choriocarcinoma
    B. Invasive Mole
    C. Placental Site Trophoblastic Tumor
    D. Epitheloid Trophoblastic Tumor
A

B. Invasive Mole

Rationale:

Invasive mole is the most common GTN following a hydatidiform mole (50% of cases).

191
Q
  1. Which GTN most commonly follows a term pregnancy or miscarriage?
    A. Choriocarcinoma
    B. Invasive Mole
    C. Placental Site Trophoblastic Tumor
    D. Epitheloid Trophoblastic Tumor
A

A. Choriocarcinoma

Rationale:

Gestational choriocarcinoma is the most common GTN following term pregnancy or miscarriage (25% of cases).
It spreads hematogenously and has early metastases (lungs, vagina, brain, liver).

192
Q
  1. Which Gestational Trophoblastic Neoplasia (GTN) develops from chorionic-type intermediate trophoblast?
    A. Choriocarcinoma
    B. Invasive Mole
    C. Placental Site Trophoblastic Tumor (PSTT)
    D. Epithelioid Trophoblastic Tumor (ETT)
A

D. Epithelioid Trophoblastic Tumor (ETT)

Rationale:

ETT originates from chorionic-type intermediate trophoblasts and mainly affects the uterus.
Findings: Irregular vaginal bleeding, low Ξ²-hCG levels, chemotherapy resistance.

193
Q
  1. A 38-year-old G2P1 (1001) is diagnosed with an invasive mole confined to the uterus. What FIGO stage is this, and what is the best treatment option?
    A. Stage I, Single-Agent Chemotherapy
    B. Stage II, Multi-Agent Chemotherapy
    C. Stage III, EMA-CO
    D. Stage IV, High-Risk Chemotherapy
A

A. Stage I, Single-Agent Chemotherapy

Rationale:

Stage I: GTN confined to the uterus.
Best treatment: Single-agent chemotherapy (Methotrexate or Actinomycin D) for low-risk cases.

194
Q
  1. Which type of hydatidiform mole has a karyotype of 69,XY?
    A. Complete Hydatidiform Mole
    B. Partial Hydatidiform Mole
    C. Invasive Mole
    D. Choriocarcinoma
A

B. Partial Hydatidiform Mole

Rationale:

Partial moles have triploid karyotypes (69,XXY, 69,XXX, or 69,XYY).
Caused by dispermic fertilization (two sperm fertilizing one egg).
Contains fetal tissue unlike complete moles.

195
Q
  1. The following are macroscopic descriptions of complete hydatidiform mole EXCEPT:
    A. Enlarged, edematous villi forming grape-like vesicles
    B. No fetal or embryonic tissue
    C. Placental tissues found in some areas of the endometrium
    D. Uterine enlargement beyond gestational age
A

C. Placental tissues found in some areas of the endometrium

Rationale:

Complete moles do NOT contain fetal or placental tissues.
The trophoblastic proliferation is diffuse, with villi appearing as clusters of vesicles.

196
Q
  1. Which of the following is a microscopic description of partial hydatidiform mole?
    A. Hydropic and normal villi
    B. Diffuse trophoblastic hyperplasia
    C. No fetal or embryonic tissue
    D. Enlarged edematous villi
A

A. Hydropic and normal villi

Rationale:

Partial moles have mixed villous populations, with both normal and hydropic villi.
Fetal tissue is present in contrast to complete moles.

197
Q
  1. Which of the following is a microscopic description of partial hydatidiform mole?
    A. Diffuse trophoblastic hyperplasia
    B. Presence of fetal tissue
    C. Hydropic and normal villi
    D. Avascular villi
A

C. Hydropic and normal villi

Rationale:

Partial moles display irregularly swollen villi, normal villi, and fetal remnants.

198
Q
  1. What are the classic features of molar pregnancy?
    A. Vaginal bleeding
    B. Hyperemesis
    C. Abdominal pain
    D. AOTA (All of the Above)
A

D. AOTA (All of the Above)

Rationale:

Heavy vaginal bleeding due to trophoblastic overgrowth.
Excessive nausea/vomiting (hyperemesis gravidarum) due to high Ξ²-hCG.
Abdominal pain due to uterine distension.

199
Q
  1. Fecal incontinence is most commonly associated with:
    A. Complete Hydatidiform Mole
    B. Partial Hydatidiform Mole
    C. Invasive Mole
    D. Choriocarcinoma
A

A. Complete Hydatidiform Mole

Rationale:

Uterine overgrowth and trophoblastic invasion may cause mass effect on the rectum.

200
Q
  1. Which GTD is associated with a triploid karyotype?
    A. Complete Hydatidiform Mole
    B. Partial Hydatidiform Mole
    C. Choriocarcinoma
    D. Placental Site Trophoblastic Tumor
A

B. Partial Hydatidiform Mole

Rationale:

Partial moles are triploid (69,XXY, XXX, or XYY), resulting from dispermic fertilization.

201
Q
  1. Which GTD lacks atypia in histology?
    A. Complete Hydatidiform Mole
    B. Partial Hydatidiform Mole
    C. Choriocarcinoma
    D. Invasive Mole
A

B. Partial Hydatidiform Mole

Rationale:

Partial moles have less trophoblastic hyperplasia, lacking atypia compared to complete moles.

202
Q
  1. Markedly elevated Ξ²-hCG is most associated with:
    A. Complete Hydatidiform Mole
    B. Partial Hydatidiform Mole
    C. Normal Pregnancy
    D. Threatened Abortion
A

A. Complete Hydatidiform Mole

Rationale:

Ξ²-hCG levels in complete moles often exceed 100,000 mIU/mL.

203
Q
  1. Diffuse trophoblastic invasion is characteristic of:
    A. Complete Hydatidiform Mole
    B. Partial Hydatidiform Mole
    C. Choriocarcinoma
    D. Placental Site Trophoblastic Tumor
A

A. Complete Hydatidiform Mole

Rationale:

Complete moles show diffuse trophoblastic hyperplasia, unlike partial moles.

204
Q
  1. Post-molar malignancy sequelae is highest in:
    A. Complete Hydatidiform Mole
    B. Partial Hydatidiform Mole
    C. Epithelioid Trophoblastic Tumor
    D. Placental Site Trophoblastic Tumor
A

A. Complete Hydatidiform Mole

Rationale:

15-20% of complete moles progress to GTN, whereas only 1-5% of partial moles do.

205
Q
  1. A 30-year-old G4P2 (2012) at 16 weeks presents with profuse vaginal bleeding. BP: 80/60, CR: 105, RR: 24. What is the best initial step?
    A. Suction curettage
    B. Stabilize the patient
    C. Perform an immediate hysterectomy
    D. Obtain a Ξ²-hCG test
A

B. Stabilize the patient

Rationale:

Stabilization (IV fluids, blood transfusion if needed) is the priority before definitive management.

206
Q
  1. A grape-like vesicular mass is found in the vaginal wall of a patient with an enlarged uterus (22 weeks AOG). What is the most likely diagnosis?
    A. Complete Mole
    B. Partial Mole
    C. Both A and B
    D. None of the above
A

A. Complete Mole

Rationale:

The classic β€œgrape-like” appearance is characteristic of complete hydatidiform mole.

207
Q
  1. What is the classic sonographic appearance of molar pregnancy?
    A. Snowstorm appearance
    B. Ground-glass appearance
    C. Double-ring sign
    D. Railroad track sign
A

A. Snowstorm appearance

Rationale:

β€œSnowstorm” sign on ultrasound is a hallmark of complete hydatidiform mole.

208
Q
  1. What is the preferred management for molar pregnancy in a 30-year-old woman?
    A. Suction Curettage
    B. Hysterectomy
    C. Expectant Management
    D. Methotrexate
A

A. Suction Curettage

Rationale:

Preferred treatment for women desiring future fertility.

209
Q
  1. What additional tests should be done in cases of molar pregnancy?
    A. CBC
    B. Chest X-ray
    C. Thyroid Function Tests (TFT)
    D. AOTA (All of the Above)
A

D. AOTA (All of the Above)

Rationale:

Chest X-ray: Detects lung metastases.
CBC: Assesses anemia from vaginal bleeding.
TFT: Screens for thyrotoxicosis secondary to high Ξ²-hCG.

210
Q
  1. What is the recommended frequency for Ξ²-hCG monitoring post-molar evacuation?
    A. Every 1-2 weeks
    B. Every 3 weeks
    C. Every month
    D. Only once after evacuation
A

A. Every 1-2 weeks

Rationale:

Weekly to biweekly Ξ²-hCG levels until they normalize, followed by monthly monitoring for 6 months.

211
Q
  1. Which malignant trophoblastic tumor consists of trimorphic proliferation of intermediate trophoblastic cells, syncytiotrophoblasts, and cytotrophoblasts?
    A. Choriocarcinoma
    B. Invasive Mole
    C. Placental Site Trophoblastic Tumor
    D. Epithelioid Trophoblastic Tumor
A

A. Choriocarcinoma

Rationale:

Choriocarcinoma is composed of syncytiotrophoblasts, cytotrophoblasts, and intermediate trophoblasts.

212
Q
  1. What is the most common clinical manifestation of Gestational Trophoblastic Neoplasia (GTN)?
    A. Vaginal bleeding
    B. Persistent nausea and vomiting
    C. Pelvic pain
    D. Dyspnea
    E. Headache
A

A. Vaginal bleeding

Rationale:

Persistent or irregular vaginal bleeding is the most common symptom of GTN.
Other symptoms include pelvic pain, enlarged uterus, and metastatic symptoms (dyspnea, neurological deficits, etc.).

213
Q
  1. What is the most common site of metastasis for GTN?
    A. Lungs
    B. Liver
    C. Brain
    D. Kidneys
A

A. Lungs

Rationale:

GTN metastasizes most commonly to the lungs (80%), followed by the vagina (30%), pelvis (20%), liver (10%), and brain (10%).

214
Q
  1. Which of the following is NOT a risk factor for choriocarcinoma?
    A. Endometriosis
    B. Prior molar pregnancy
    C. History of miscarriage
    D. Advanced maternal age
    E. Prior term pregnancy
A

A. Endometriosis

Rationale:

Choriocarcinoma arises from trophoblastic tissue, not from endometrial pathology like endometriosis.
Risk factors include prior molar pregnancy, miscarriage, term pregnancy, and advanced maternal age.

215
Q
  1. Which GTN occurs in reproductive-age women?
    A. Gestational Choriocarcinoma
    B. Placental Site Trophoblastic Tumor (PSTT)
    C. Epithelioid Trophoblastic Tumor (ETT)
    D. All of the above
A

D. All of the above

Rationale:

All GTNs can occur in reproductive-age women, though PSTT and ETT are rarer than choriocarcinoma.

216
Q
  1. GTN that occurs after a previous pregnancy?
    A. Gestational Choriocarcinoma
    B. Placental Site Trophoblastic Tumor (PSTT)
    C. Epithelioid Trophoblastic Tumor (ETT)
    D. All of the above
A

D. All of the above

Rationale:

GTN can occur after any pregnancy, including molar pregnancy, miscarriage, or term pregnancy.
50% of GTN follows molar pregnancy, 25% follows miscarriage, 25% follows normal pregnancy.

217
Q
  1. Which GTN has vaginal bleeding as a clinical presentation?
    A. Gestational Choriocarcinoma
    B. Placental Site Trophoblastic Tumor (PSTT)
    C. Epithelioid Trophoblastic Tumor (ETT)
    D. All of the above
A

D. All of the above

Rationale:

Vaginal bleeding is a common feature of all GTNs, as they invade the endometrium and myometrium.

218
Q
  1. Which GTN is associated with markedly increased serum Ξ²-hCG?
    A. Gestational Choriocarcinoma
    B. Placental Site Trophoblastic Tumor (PSTT)
    C. Epithelioid Trophoblastic Tumor (ETT)
    D. All of the above
A

A. Gestational Choriocarcinoma

Rationale:

Gestational choriocarcinoma has the highest Ξ²-hCG levels due to its aggressive trophoblastic proliferation.
PSTT and ETT have lower Ξ²-hCG levels than choriocarcinoma.

219
Q
  1. Which GTN has a growth pattern of large tumor masses?
    A. Gestational Choriocarcinoma
    B. Placental Site Trophoblastic Tumor (PSTT)
    C. Epithelioid Trophoblastic Tumor (ETT)
    D. All of the above
A

D. All of the above

Rationale:

All three GTNs can form large tumor masses due to uncontrolled trophoblastic growth.

220
Q
  1. Which GTN is associated with mild cytologic atypia?
    A. Gestational Choriocarcinoma
    B. Placental Site Trophoblastic Tumor (PSTT)
    C. Epithelioid Trophoblastic Tumor (ETT)
    D. All of the above
A

B. Placental Site Trophoblastic Tumor (PSTT)

Rationale:

PSTT shows mild cytologic atypia compared to choriocarcinoma, which exhibits more pleomorphism and hyperchromasia.

221
Q
  1. Which GTN is most commonly located in the corpus of the uterus?
    A. Gestational Choriocarcinoma
    B. Placental Site Trophoblastic Tumor (PSTT)
    C. Epithelioid Trophoblastic Tumor (ETT)
    D. All of the above
A

D. All of the above

Rationale:

All three GTNs can involve the uterine corpus, but PSTT and ETT are more locally invasive.

222
Q
  1. FIGO staging of GTN is based on:
    A. Anatomical staging
    B. Histological classification
    C. Hormonal response
    D. Tumor marker levels
A

A. Anatomical staging

Rationale:

FIGO GTN staging is based on anatomical spread of disease (uterus, vagina, lungs, distant metastases).

223
Q
  1. The WHO Prognostic Scoring System for GTN, as adopted by FIGO, is used to:
    A. Determine the risk for drug failure
    B. Identify the type of chemotherapy to use
    C. Differentiate benign from malignant GTN
    D. Determine the histological subtype
A

A. Determine the risk for drug failure

Rationale:

The WHO Prognostic Scoring System predicts the likelihood of chemotherapy resistance.
Scores ≀6 are low risk (single-agent therapy), whereas scores β‰₯7 are high risk (multi-agent chemotherapy needed).